LE Mock17
LE Mock17
Part of the Most Comprehensive & Consistently Successful Study Material & Test Series Module, spanning across
both Physical and Online Programs in the entire Country. As a result, LegalEdge was able to engineer Clean-Sweep-
Landslide figures of a handsome 64 Selections under 100 ranks, and a whopping 273 selections under 500 ranks in
CLAT 2021. With AILET being no different, a total of 34 of our students found their way into NLU, Delhi in 2021.
In a nutshell, every second admit in a Top National Law School in 2021 came from the LegalEdge Preparation
Ecosystem.
1. No clarification on the question paper can be sought. Answer the questions as they are.
2. There are 150 multiple choice objective type questions.
3. There is negative marking of 0.25 for every incorrect answer. Each question carries ONE mark. Total marks
are 150
4. You have to indicate the correct answer by darkening one of the four responses provided, with a BALL PEN
(BLUE OR BLACK) in the OMR Answer Sheet.
Example: For the question, "Where is the Taj Mahal located?", the correct answer is (b).
The student has to darken the corresponding circle as indicated below:
(a) Kolkata (b) Agra (c) Bhopal (d) Delhi
Right Method Wrong Methods
5. Answering the questions by any method other than the method indicated above shall be considered incorrect
and no marks will be awarded for the same.
6. More than one response to a question shall be counted as wrong.
7. Do not write anything on the OMR Answer Sheet other than the details required and, in the spaces, provided
for.
8. You are not required to submit the OMR Answer Sheet and Test Paper after the test.
9. The use of any unfair means by any candidate shall result in the cancellation of his/her candidature.
10. Impersonation is an offence and the student, apart from disqualification, may have to face criminal
prosecution.
11. You have to scan the QR code only after completion of offline test.
12. You cannot leave the examination hall without punching your answers on the portal.
SECTION-A : ENGLISH LANGUAGE
Passage (Q.1-Q.5): Read the following passages carefully and answer the questions given below it.
Paragraph 1: The government – and Paytm – may not agree, but there are some downsides to the rising
digitization and connectivity. One is an unleashing of aspirations. Everyone wants not just what Bengal’s
leftists used to contemptuously dismiss as components of the middle-class Indian dream — gaadi, baadi,
chaakri (car, home, job) — but a whole lot of other things. From watching the latest Salman-starrer now —
which is why India is the ‘download’ capital of the world — to the latest phone and footwear, people desire
for all forms of comfort. This is a downside because the Indian economy is simply not in a position to create
the kind of ecosystem which will enable all these aspirations to be realized by a vast majority of the people.
The other downside is the near-instantaneous transmission of unrest. Thanks to YouTube and WhatsApp,
an expression of farmer unrest can travel from Mannargudi to Mandsaur faster than any ponderous
government’s attempts to address or contain the disturbance within a particular area.
Paragraph 2: If we haven’t had more such demonstrations, or bigger unrest, it is probably because there
hasn’t been an issue big enough to concern all the people across the country at the same time. The recent
farmer protests, big and significant though they were, left urban India untouched. But there is one issue
which, sooner or later, may unleash social unrest on a scale which will make the farmer agitation look like
a toddler’s tantrum — jobs. Or more precisely, the lack of them. That the available workforce of a country
needs to be occupied in fruitful employment is a no-brainer. The reason global businesses are investing
billions of dollars in India is its potential to become one of the world’s economic powerhouses.
Paragraph 3: This potential derives from India’s ‘demographic dividend’ — the millions of young people
joining the workforce every year. India will add more than 100 million people of working age between now
and 2025, by which time it will account for one-fifth of the entire world’s workforce. That creating productive
and remunerative jobs for these aspirants is India’s biggest economic challenge is also a no-brainer.
Paragraph 4: If that be the case, then, going by the BSE’s own yardstick, India appears to be in a very
good place. If 96% of India’s workforce is productively employed, as the index suggests, then Prime
Minister Narendra Modi has more than made good on his pre-poll promise of creating one crore new jobs
in five years. It also means that India is now well and truly a middle-income country, with poverty and
deprivation confined to isolated pockets, and many, if not all, in a position to fulfill most of those newly
unleashed aspirations.
Paragraph 5: Economist Sudipto Mundle argued in a recent paper that “the vast majority of India’s working
households are still living precariously on the brink of survival”. This is Modi’s — and India’s — biggest
challenge. It is not just about creating jobs but generating employment that yields substantially more than
mere sustenance. Otherwise, the ‘aspirational young India’ will turn into an ‘angry young India’.
1. According to the first paragraph, which of the following is untrue?
(a) Everything is not hunky dory in the digitization projection in India.
(b) A middle class Indian dreams about having a good job, affordable housing and a sensible car to drive
around.
(c) Salmaan starrers are the only ones which are being downloaded by the Indians as part of digitization.
(d) Indian economy needs to pass through various upward modes to cater to the aspirations of its major
chunk of population.
2. In the second paragraph, the author uses the phrase 'a toddler's tantrum'. Which of the following reflects
the best contextual meaning of the phrase?
(a) The farmers' agitation is still in the infant stage but may become serious in future.
(b) When compared to the looming job situation, the impact of the farmers' agitation was a soft fall.
(c) Agitating for rights is child's play for the farmers as they have been receiving exclusive coverage on the
social media.
(d) Farmers' agitation is like a child's tantrum. It subsides on its own after a while.
Head Office: 127, Zone II, MP Nagar, Bhopal |+91-7676564400| https://www.toprankers.com Page 2 of 36
3. The third paragraph talks of a 'demographic dividend'. What does it imply in the context of the passage?
(a) When the world over is facing the problem of an aging population, India has a young human resources
capital.
(b) The population boom in India has been successfully contained.
(c) The B-schools in India are turning out many young people every year adding to the bleak job market.
(d) There is a vast gap between the aging and young population in India.
4. Which of the following statements connects paragraph three with paragraph four?
(a) According to popular adage, a country can be called as zero unemployed if every citizen is engaged in
one work or the other.
(b) Dattu is a laborer earning Rs. 5000 a month, barely managing his family needs and is gainfully
employed.
(c) By 2025, Prime Minister Narendra Modi will be in a position to declare that 96 percent of the workforce
as employed productively, in India.
(d) The age-old adage of 'garibi hatao' will become true in India in 2025.
5. What is the message of caution given by the author in the fifth paragraph?
(a) Indian youth is very restless and may turn angry if not provided with jobs of their choice.
(b) Poverty can never be eradicated in India.
(c) Prime Minister Narendra Modi needs to work very hard to get rid of poverty in India.
(d) India accounting for not just sustainable, but a more productive and remunerative jobs, by 2025 is just
a bubble and needs more introspection.
Passage (Q.6-Q.10): Our planet's natural ecosystems are in trouble. Recent advances in "big data" and
improved remote sensing tools show us that collective human impacts are leaving fewer places untouched,
with only 15% of the Earth's land mass formally protected and global biodiversity declining at an
unprecedented rate.
Global assessments led by scientists, such as the 5th Global Biodiversity Outlook published this week, and
others endorsed by governments through bodies like the Intergovernmental Panel on Biodiversity and
Ecosystem Services (IPBES), make it clear that governments are failing to meet existing global targets for
biodiversity, and that critical ecosystems like coral reefs will be altered to the point that the biodiversity they
hold, and the services they provide, will be damaged beyond repair.
Corals in particular have shown the most rapid increase in extinction risk of all assessed species groups,
with studies showing an estimated two thirds of coral reef fish lost compared to historical reefs as only 2.5
percent of the world's reefs are being actively protected. We must respond to this remarkable scientific and
political consensus on biodiversity loss with meaningful action. As an international community, we must do
more to help protect those important ecosystems such as coral reefs, which provide extraordinary
contributions to both biodiversity (about 25 percent of all marine biodiversity across about 0.1 percent of
the ocean floor) and human wellbeing (economic and food security for hundreds of millions of people).
International plans, usually in the form of policy frameworks, are well suited to globalized threats such as
climate change, but have a hard time addressing the myriad complex localized threats facing coral reefs
(such as overexploitation, pollution, coastal and industrial development) and other ecosystems.
They do, however, play an important role in driving consensus around how to measure and monitor
ecosystems like coral reefs and the benefits they provide to people. They also galvanise political will, unlock
billions in financing for nature conservation, and provide the impetus to drive change at a local level.
Currently, all but a few of the world’s governments are in the process of negotiating updated, consensus-
based goals and targets for biodiversity conservation to replace those that expire in 2020. This “post -2020
global biodiversity framework” will guide the action and investments of the 196 governments that are party
to the Convention on Biological Diversity (CBD). The negotiations are expensive and complex. Over the
last two years, members of the International Coral Reef initiative (ICRI), a global partnership of more than
forty governments and more than forty civil society organization with coral reef expertise, have agreed on
this framework that are critical for coral reefs.
Head Office: 127, Zone II, MP Nagar, Bhopal |+91-7676564400| https://www.toprankers.com Page 3 of 36
6. Which of the following reflects the complexity of the international plans for ecosystem problems?
(a) The plans are usually in the form of policy framework.
(b) Governments don't follow the policy framework for a longer duration.
(c) They play an important role in driving the consensus around how to measure and monitor Ecosystems.
(d) International plans have a hard time tackling the multitude and complex localized menaces facing coral
reefs and other ecosystems.
7. In the sentence, "As an international community, we must do more to help protect those important
ecosystems", Which important ecosystems are being referred to?
(a) Marine ecosystems.
(b) Only coral reefs.
(c) Ecosystems which provide contribution to human wellbeing.
(d) Ecosystems which provide contribution to human wellbeing and biodiversity.
10. As per the passage, which of the following sentences is not true?
(a) The natural ecosystem of earth is in trouble.
(b) The existing global targets for biodiversity are not being met.
(c) Members of the International Coral Reef Initiative have agreed on the steps we must take.
(d) Recent advances in "big data" have helped in the decline of global biodiversity.
Passage (Q.11-Q.15): One core component of the Belt and road Initiative (BRI) is the BeiDou Navigation
Satellite System (BDS). BDS, while largely a benign satellite navigation system, has a key component of
entrapment for economically weaker countries. What exactly are the malign components of the BDS that
have the potential to entrap countries in technology dependence on Beijing?
The benign parts of BDS can be gauged by the fact that the linked services are either replaceable, or do
not entail a continuing use of BDS past the implementation-construction phase. To foster digital economies
in Arab states, Beijing established a BDS in Tunis. This involved the training Arab state representatives in
transitioning to digital economies, and said training was done on the basis of BDS to show them the
opportunities and benefits.
For a long time, services we take for granted - like GPS navigation - were dependent on the US-controlled
Navstar satellites whose services were prohibited from being extended to Iran. A Memorandum of
Understanding (2015) between China and Iran resolved these issues with BeiDou, providing high-precision
navigation services, and spawned a host of services based on this technology. In South Asia, Pakistan's
new Islamabad International Airport's information integration system offers precise time synchronisation
and coordinated operation of the airport's sub-systems in a "reliable" manner.
Myanmar, Laos, Cambodia, Indonesia, Thailand, and Uganda have used BDS for agricultural and
infrastructural purposes. These include land, water, and ecology surveys. They have been able to benefit
by planning their crop cycles and water allocation with great precision, resist climatic variation, and increase
the accuracy of their national disaster agencies through dependence on these services. In some of these
countries, like Thailand, power distribution, and goods delivery and tracking are all linked to BDS.
Head Office: 127, Zone II, MP Nagar, Bhopal |+91-7676564400| https://www.toprankers.com Page 4 of 36
On the face of it, all these above-mentioned projects seem benign, with only benefits to be reaped by the
countries dependent on the system. Yet, each of these associations has masked a critical element of
dependence. Today, Iran's entire basket of satellite-based services - their equivalents of Google Maps,
Uber, Zomato, Amazon, etc. - and the massive economic footprint they have spawned are dependent on
BDS. Any withdrawal of services would be catastrophic for large parts of the country's urban economy that
depend heavily on this system.
In a similar manner, agricultural systems in several parts of Southeast Asia, as discussed earlier, have
been enhanced based on BDS technology. Sustaining this expansion is also based on BDS, as it is used
to plan cropping seasons, adjust water distribution, and compensate for climate. Indeed, once even basic,
and traditional activities like agriculture become technology-dependent, their extraordinary increase in
productivity and resistance to climactic variation also depend on the usage of this technology.
11. Which of the following is referred to the 'critical element of dependence' as mentioned in the given passage?
(a) The expansion of the BDS Technology and the increase in the number of countries who accept it.
(b) The utter dependence on the BDS technology, from where going back is not possible and any departure
from the services of the technology would be disastrous.
(c) The extraordinary increase in productivity and resistance leading to the betterment of the agricultural
system.
(d) The transfer of the technology from Navstar satellites to BeiDou Navigation Satellite System in most
Asian countries.
12. What is the author's motive behind writing the fifth paragraph of the given passage?
(a) To tell the reader how BDS has the ability to entrap countries in technology dependence.
(b) To tell the reader about overall dependence of Iran on China's various technologies.
(c) To explain about the pros and cons of BDS for Iran.
(d) To tell the reader about different satellite-based services in Iran.
13. Which of the following is similar in meaning to the word 'benign' as mentioned in the given passage?
(a) Meticulous (b) Propitious (c) Unerring (d) Solemn
14. With which of the following is the author least likely to agree?
(a) There are many benefits of the BDS technology.
(b) Iran itself is responsible for its own course of action.
(c) BDS technology will eventually cause trouble to the whole world.
(d) The BDS technology largely focus on poor countries.
Passage (Q.16-Q.20): Perhaps no other social entity appears more 'natural' than the family. Often, we are
prone to assume that all families are like the ones we live in. No other social institution appears more
universal and unchanging. Sociology and social anthropology have over many decades, conducted field
research across cultures to show how the institutions of family, marriage and kinship are important in all
societies and yet their character is different in different societies. They have also shown how the family (the
private sphere) is linked to economic, political, cultural and educational (the public) spheres.
According to the functionalists the family performs important tasks, which contribute to society's basic
needs and helps perpetuate social order. The functionalist perspective argues that modern industrial
societies function best if women look after the family and men earn the family livelihood. In India studies,
Head Office: 127, Zone II, MP Nagar, Bhopal |+91-7676564400| https://www.toprankers.com Page 5 of 36
however, suggest that families need not become nuclear in an industrial pattern of economy. This is but
one example to show how trends based on experiences of one society cannot necessarily be generalised.
The nuclear family is seen as the unit best equipped to handle the demands of industrial society by the
functionalists. In such a family one adult can work outside home while the second adult cares for the home
and children. In practical terms, this specialisation of roles within the nuclear family involves the husband
adopting the 'instrumental' role as breadwinner, and the wife assuming the 'affective', emotional role in
domestic settings (Giddens 2001). This vision is questionable not just because it is gender unjust but
because empirical studies across cultures and history show that it is untrue.
A central debate in India has been about the shift from nuclear family to joint families. We have already
seen how sociology questions common sense impressions. The fact is that nuclear families have always
existed in India particularly among deprived castes and classes.
The sociologist A.M. Shah remarks that in post-independent India, the joint family has steadily increased.
The contributing factor is the increasing life expectancy in India according to him. It has increased from
32.5 - 55.4 years for men and from 31.7 - 55.7 years for women during the period 1941 - 50 to 1981- 85.
Consequently, the proportion of aged people (60 years and above) in the total population has increased.
This again is a broad generalisation. But in the spirit of the sociological perspective, it cautions us against
blindly believing a common-sense impression that the joint family is fast eroding. And alerts us to the need
for careful comparative and empirical studies.
16. Which of the following strengthens the author's argument that the character of family, marriage and kinship
is different in different societies?
(a) Importance of family, marriage and kinship is different in different societies.
(b) The private sphere such as family is linked to economic, political, cultural, and educational spheres.
(c) Family plays an important role in a person's life and give strength to the individual irrespective of the
societies.
(d) Marriage as an institution is as successful in western culture as much as it is in the Indian culture.
17. Which of the following is similar in meaning to the word 'perpetuate' as mentioned in the passage?
(a) Discern (b) Halt (c) Cease (d) Sustain
18. According to the author, the division of roles within the nuclear families is
(a) Justified, but a few sections of the society are against it.
(b) Is not true as per the empirical studies across cultures.
(c) Is not according to the demands of industrial society.
(d) Is a myth with respect to the western societies.
19. According to the passage, why was there a steady increase in joint family in the post-independent era?
(a) Increase in life expectancy in India.
(b) Increase in the proportion of younger people.
(c) Decrease in the number of aged people.
(d) Increase in the number of nuclear family.
20. What is the author's purpose behind writing the first paragraph of the given passage?
(a) To realise the importance of family and how it is affected by the economic and cultural spheres.
(b) To reveal the changing principles of family as joint families are seen very often.
(c) To highlight how the universal and important social institution i.e., family can vary as per its public
sphere.
(d) To indicate that all the societies are not same and do not have the same set of values.
Head Office: 127, Zone II, MP Nagar, Bhopal |+91-7676564400| https://www.toprankers.com Page 6 of 36
Passage (Q.21-Q.25): The island of Ireland, whose northernmost part lies a mere 13 miles from Britain,
has been contested territory for at least nine centuries. Britain long gazed with colonial ambitions on its
smaller Catholic neighbour. The 12th-century Anglo-Norman invasion first brought the neighbouring
English to Ireland.
In the late 16th century, frustrated by continuing native Irish resistance, Protestant England implemented
an aggressive plan to fully colonize Ireland and stamp out Irish Catholicism. Known as “plantations”, this
social engineering exercise “planted” strategic areas of Ireland with tens of thousands of English and
Scottish Protestants.
Plantations offered settlers cheap woodland and bountiful fisheries. In exchange, Britain established a base
loyal to the British crown – not to the Pope.
England’s most ambitious plantation strategy was carried out in Ulster, the northernmost of Ireland’s
provinces. By 1630, according to the Ulster Historical Foundation, there were about 40,000 English-
speaking Protestant settlers in Ulster.
Though displaced, the native Irish Catholic population of Ulster was not converted to Protestantism.
Instead, two divided and antagonistic communities – each with its own culture, language, political
allegiances, religious beliefs and economic histories – shared one region. Over the next two centuries,
Ulster’s identity divide transformed into a political fight over the future of Ireland.
“Unionists” – most often Protestant – wanted Ireland to remain part of the United Kingdom. “Nationalists” –
most often Catholic – wanted self-government for Ireland. These fights played out in political debates, the
media, sports, pubs – and, often, in street violence. By the early 1900s, a movement of Irish independence
was rising in the south of Ireland. The nationwide struggle over Irish identity only intensified the strife in
Ulster.
The British government, hoping to appease nationalists in the south while protecting the interests of Ulster
unionists in the north, proposed in 1920 to partition Ireland into two parts: one majority Catholic, the other
Protestant-dominated – but both remaining within the United Kingdom. Irish nationalists in the south
rejected that idea and carried on with their armed campaign to separate from Britain. Eventually, in 1922,
they gained independence and became the Irish Free State, today called the Republic of Ireland.
In Ulster, unionist power-holders reluctantly accepted partition as the best alternative to remaining part of
Britain. In 1920, the Government of Ireland Act created Northern Ireland, the newest member of the United
Kingdom.
In this new country, native Irish Catholics were now a minority, making up ___(A)___ of Northern Ireland’s
1 .2 million people. Stung by partition, nationalists refused to recognise the British state. Catholic
schoolteachers, supported by church leaders, refused to take state salaries. And when Northern Ireland
seated its first parliament in May 1921, nationalist politicians did not take their elected seats in the
assembly. The Parliament of Northern Ireland became, essentially, Protestant – and its proBritish leaders
pursued a wide variety of anti-Catholic practices, discriminating against Catholics in public housing, voting
rights and hiring.
21. Which of the given options can be used to complete the blank labelled (A) in the most appropriate way,
contextually and grammatically?
(a) less by a third
(b) less by one thirds
(c) less than a thirds
(d) less than a third
Head Office: 127, Zone II, MP Nagar, Bhopal |+91-7676564400| https://www.toprankers.com Page 7 of 36
22. Which of the following statement (s) is/are NOT TRUE in accordance with the information provided in the
passage?
I. Britain successfully suppressed Catholicism in Ireland.
II. The British crown was under authority of the Pope.
III. "Plantations" restructured the society of Northern Ireland.
(a) Only I (b) Only III (c) Both II & III (d) Both I & II
23. Which of the following is/are correctly inferred from the given passage?
I. Ulster was chosen as a plantation because of its geographical significance.
II. The people of Ulster actively took part in the conflict.
III. Ulster unionists didn't support the formation of Irish Free State.
(a) Only I (b) Only III (c) Both II & III (d) Both I & II
24. Which of the following words as used in the passage can be replaced by the word “belligerent”?
(a) Strife (b) Appease (c) Gazed (d) Aggressive
Passage (Q.26-Q.30): A murmur from the driver’s seat indicated that I was near my destination. From the
confines of the car I scanned the dark surroundings with eyes heavy with sleep. Kochi’s noisy, unruly
evening traffic was around 55 km behind us. I attributed my sluggishness to the near-silence that rules
Thattekad, a bird sanctuary in the foothills or the Western Ghats in Kerala’s Ernakulam district.
My eyes were half-closed as I entered my room at the home stay. The lone photograph in the room - that
of three unattractive birds captioned as Ceylon frog mouth - did little to perk me up. I remained in this state
until dinner summons. Frog mouth photos stared back from every wall in the house as I went through the
meal in a haze. By the time I climbed into bed, I’d had an overdose of the nocturnal grey-brown bird. Or so
I thought.
The morning arrived early. Girish, the guide, took our group of three (two birding fanatics from Bengaluru
being my companions for the day) into the forest. We walked under a canopy of teak, mahogany, and
bamboo trees, at a rocky patch. Girish instructed us to look out for the black baza, a small bird of prey. The
baza didn’t oblige but many others did. In a span of 30 minutes, we spotted the Asian brown flycatcher, the
orange headed thrush, the Oriental honey buzzard and a variety of woodpeckers.
The little black raptor chose to stay elusive. In a desperate bid to divert attention from the bird that remained
invisible, I voiced my ‘desire’ to see the unsightly frog mouth. We moved ahead in search of the new target.
But instead of the grey-brown plumage of the peninsular bird, we were rewarded with the eye-popping
colors of the Malabar trogon. And before we knew it, the canopy overhead had given way to the most
spotless blue sky. We reached a water body in the middle of the sanctuary. A handful of dead tree trunks
jutted out of the water. It was the end of the walk and I was happy to just sit by and absorb the stillness
around.
The frog mouth had been declared an endangered species in 1983. Improving their numbers proved difficult
given that the bird lays only one egg on an average every year. More than two decades on, the number of
frog mouths at Thattekad has climbed close to 50 pairs.
My quest for the bird I had begun to admire continued over the next day. We drove towards Pandapara,
where the frog mouth had been sighted close to where we stood. We waded through the knee-deep waters
of a stream before we arrived at another rocky patch. A few minutes later he pointed to a tree to the left of
the trail we were on. There, in perfect camouflage with the dead leaves, sat a pair of frog mouths. If one of
them hadn’t blinked I’d have assumed those birds were stuffed!
Head Office: 127, Zone II, MP Nagar, Bhopal |+91-7676564400| https://www.toprankers.com Page 8 of 36
26. In the beginning of the passage the author is not very keen on the photograph of the frog mouth, but the
next day he is desperate to spot it. Why?
(a) The author is an ornithologist out on bird spotting.
(b) He was tired after the journey and the unattractive snaps of the frog mouth did not help.
(c) The frog mouth was an endangered species and spotting it would be any ornithologist’s dream.
(d) All the above
27. Why does the author call his friends ‘a couple of birding fanatics?’.
(a) They were fanatic about birds and their habitat.
(b) The author and his friends are ornithologists, and they are so committed to their profession that they
are passionate about spotting birds.
(c) The author’s friends were interested only in birds and could even skip food for their sake and the author
finds it difficult to keep pace with them.
(d) The authors friends were ardent bird lovers and kept trekking fanatically till they spotted the bird they
wished to see.
28. What does the author mean when he says ‘absorbing the stillness around’?
(a) Sitting absolutely still so as not to disturb the birds.
(b) After a long walk in the jungle, the author is just relaxing and stretching his feet.
(c) The author is an ornithologist and hence is always attracted by nature. Here, he just sits still feeling
one with the nature.
(d) It is an expression to denote that the scene with the water body was so ethereal that the author forgot
himself and sat still.
29. Frog is an amphibian and the frog mouth belongs to the avian community. What can be common between
them?
(a) Both have descended from the frog.
(b) Both eat similar types of insects.
(c) The frog mouth gapes like a frog.
(d) Both have the same ancestors.
30. ‘If one of them hadn’t blinked I’d have assumed those birds were stuffed!’ This is the last line of the passage.
What do you infer from this?
(a) As the frog mouth was endangered the author could only see the stuffed version of the birds.
(b) The frog mouths blended so beautifully with the foliage and sat so motionlessly that only their blinking
could differentiate them.
(c) The author was waiting for the frog mouths to blink so that he could capture them in his camera.
(d) The frog mouths were so scared with the human treads that they even forgot to blink.
Head Office: 127, Zone II, MP Nagar, Bhopal |+91-7676564400| https://www.toprankers.com Page 9 of 36
SECTION-B : GENERAL KNOWLEDGE/CURRENT AFFAIRS
Directions(Q.31-Q.65): Read the information carefully and answer the questions based on it.
Passage (Q.31-Q.35): Prime Minister Narendra Modi on Monday launched the Indian Space Association
(ISpA), an industry body consisting of various stakeholders of the Indian space domain. The members of
the organisation include government bodies such as Indian Space Research Organisation (ISRO) and
private telecom companies such as Bharti Airtel’s One Web, Tata Group’s Nelcom, L&T, MapMyIndia, and
others.
Ever since the race to reach the space and then land on the Moon began between the US and the erstwhile
USSR, governments across the world have poured millions of dollars to push the envelope in term of
exploring the edges of the space. With time, governments and government agencies collaborated to
explore newer planets and galaxies in search of life forms that exist outside Earth.
In the recent past, private sector companies such as Elon Musk’s SpaceX, Richard Branson’s Virgin
Galactic, and Jeff Bezos’ Blue Origin have taken the lead in spaceflight, promising to start tourist flights to
space.
31. Which of the following is not one of the pillars of Government's approach to space reforms?
(a) freedom of innovation to the private sector.
(b) the role of the government as an enabler.
(c) preparing youth for the future.
(d) to lead the space race in the world.
32. Which of the following is not correct about the Indian Space Association?
(a) ISpA aspires to be the collective voice of the Indian Space industry.
(b) ISpA will be represented by leading domestic and global corporations that have advanced capabilities
in space and satellite technologies.
(c) One of the main goals of the organisation is to supplement the government’s efforts towards making
India a global leader in commercial space-based excursions.
(d) ISpA will enable the government to take lead in the space projects instead of letting the private sector
to lead.
33. Which of the of the following was established by the Government to act as the Marketing arm of ISRO?
(a) Antrix
(b) Indian National Space Promotion and Authorization Centre
(c) New India Limited
(d) New space India limited
34. Indian national committee for Space research was established under the Prime Ministership of which
among the following?
(a) Jawaharlal Nehru (b) Lal Bhadur Shahtri
(c) Indira Gandhi (d) Gulzarilal Nanda
35. Which among the following is the first satellite launched by ISRO?
(a) Aryabhata (b) Bhaskara Sega I
(c) Rohini RS I (d) INSAT 1A
Head Office: 127, Zone II, MP Nagar, Bhopal |+91-7676564400| https://www.toprankers.com Page 10 of 36
Passage (Q.36-Q.40): A majority of the world’s nations have signed a historic pact that could force
multinational companies to pay their fair share of tax in markets where they operate and earn profits. One
hundred and thirty-six countries, including India, agreed Friday to enforce a minimum corporate tax rate of
[1], and an equitable system of taxing profits of big companies in markets where they are earned.
The move is part of an evolving consensus that big multinationals are funnelling profits through low-tax
jurisdictions to avoid paying taxes. The [2], comprising mostly developed economies, has led talks on a
minimum corporate tax rate for a decade. A multilateral convention is to be signed next year.
The biggest impact is likely on Big Tech companies that have largely chosen low-tax jurisdictions to
headquarter their operations.
36. Which of the following would replace [1] in the above passage?
(a) 30% (b) 25% (c) 20% (d) 15%
38. Which of the following Countries has not agreed to the deal?
(a) Kenya (b) Pakistan (c) Sri Lanka (d) All of the above
39. Which of the following will replace [2] in the above passage?
(a) Organisation for Economic Cooperation and Development
(b) World Bank
(c) International Monetary Fund
(d) Asian Development Bank
Passage (Q.41-Q.45): The Supreme Court has declared the National Green Tribunal’s (NGT) position as
a “unique” forum endowed with suo motu powers to take up environmental issues across the country.
Suo motu power was not conferred under the NGT Act, and the specialised tribunal had to be moved by
an outside party for it to act.
But the judges noted that it would be procedural hair-splitting to argue that the NGT could act upon a letter
being written to it, but learning about an environmental exigency through any other means cannot trigger
the NGT into action.
To endorse such an approach would surely be rendering the forum procedurally shackled or incapacitated,
a three-judge Bench of Justices A.M. Khanwilkar, Hrishikesh Roy and C.T. Ravikumar has said in their
judgement.
41. India became the ______ Country which has a Specialised Tribunal for Environment protection.
(a) 1st (b) 2nd (c) 3rd (d) 4th
42. NGT Act was passed by the Parliament in which of the following years?
(a) 2008 (b) 2010 (c) 2013 (d) 2015
Head Office: 127, Zone II, MP Nagar, Bhopal |+91-7676564400| https://www.toprankers.com Page 11 of 36
43. Which of the following Statement is incorrect?
(a) NGT is mandated to make disposal of applications or appeals finally within 12 months of filing the same.
(b) The NGT has five places of sittings, New Delhi is the Principal place of sitting and Bhopal, Pune, Kolkata
and Chennai are the other four.
(c) The Tribunal is headed by the Chairperson who sits in the Principal Bench and has at least ten but not
more than twenty judicial members and at least ten but not more than twenty expert members.
(d) The functional capacity of the NGT is intended to leverage wide powers to do full justice in its
environmental mandate.
44. Where and in how many days can an appeal to the decision of NGT can be filed?
(a) High Court and 90 days (b) Supreme Court and 90 days
(c) High Court and 60 days (d) Supreme Court and 60 days
45. Under which article of Constitution of India, the decision of Supreme Court of India is binding on all Court
within the territory of India?
(a) Article 124 (b) Article 129 (c) Article 136 (d) Article 141
Passage (Q.46-Q.50): Disparities in multidimensional poverty among ethnic groups are consistently high
across many countries and in nine ethnic groups more than 90 percent of the population is trapped in
poverty, according to new analysis on global multidimensional poverty released today.
The Global Multidimensional Poverty Index (MPI) produced by the United Nations Development
Programme (UNDP) and the [1] measures poverty by considering various deprivations experienced by
people in their daily lives, including poor health, insufficient education and a low standard of living. Today’s
report examines the level and composition of multidimensional poverty across 109 countries covering 5.9
billion people and presents an ethnicity/race/caste disaggregation for 41 countries with available
information.
47. Which of the following statements is INCORRECT according to the MPI report?
(a) 1.3 billion people are multidimensionally poor.
(b) About one-third (644 million) are children under age 18.
(c) Nearly 85% live in Sub-Saharan Africa (556 million) or South Asia (532 million).
(d) More than 67% live in middle-income countries.
48. Which of the following countries has been reported to show fastest periodic reduction in MPI?
(a) Sierra Leone (b) Seychelles (c) Madagascar (d) Burundi
Head Office: 127, Zone II, MP Nagar, Bhopal |+91-7676564400| https://www.toprankers.com Page 12 of 36
Passage (Q.51-Q.55): The Supreme Court on Thursday asked the government to explain how it zeroed in
on the figure of ‘[1]’ as the annual income criterion to identify Economically Weaker Sections (EWS) among
forward classes of society for grant of 10% reservation in medical admissions under the all India quota
(AIQ).
The Supreme Court’s query is significant as the 103rd Constitutional Amendment of 2019, which introduced
the 10% EWS quota, is itself under challenge before a larger Bench. The amendment is under question for
making economic criterion as the sole ground for grant of reservation benefits.
The court was hearing a batch of petitions filed by NEET aspirants challenging a July 29 notification of the
Centre announcing 27% quota to OBCs and 10% reservation to EWS in the all India quota category.
52. The 10% EWS quota was introduced under the 103rd Constitution (Amendment) Act, 2019 by amending
which of the following Articles?
(a) Articles 18 and 20 (b) Articles 14 and 22
(c) Articles 15 and 16 (d) Articles 33 and 34
53. The aforementioned amendments run contrary to the judgement pronounced in which of the following
cases?
(a) Ashoka Kumar Thakur v. Union of India (b) Maneka Gandhi v. Union of India
(c) Indra Sawhney v. Union of India (d) Shreya Singhal v. Union of India
54. Mandal Commission for reservation to the OBCs was formed under which Prime Minister?
(a) Morarji Desai (b) Indira Gandhi (c) V.P. Singh (d) P.V. Narsimha Rao
55. In July 2021, which of the following states became the first state in the country to provide one percent
reservation for the transgender community in all government services?
(a) Maharashtra (b) Tamil Nadu (c) Odissa (d) Karnataka
Passage (Q.56-Q.60): The Calcutta High Court has given West Bengal Assembly Speaker Biman Banerjee
a deadline of Thursday, October 7 to pass an order in the defection case involving MLA Mukul Roy. He
had contested and won the 2021 Assembly elections on a BJP ticket and then joined the Trinamool
Congress. BJP MLA Suvendu Adhikari, Leader of Opposition in the Assembly, has petitioned the Speaker
to disqualify Roy and two other BJP MLAs for joining the Trinamool Congress. These petitions are under
the [1] of the Constitution, i.e. the anti-defection law.
Anti-defection proceedings are also going on in other states. In Jharkhand, former CM Babulal Marandi
faces such proceedings after merging his party, Jharkhand Vikas Morcha (Prajatantrik), with the BJP. In
Rajasthan, six Bahujan Samaj Party (BSP) MLAs have merged their legislature party with the ruling
Congress, a move challenged by the BSP, and the Supreme Court recently gave the six MLAs a final
opportunity to explain the merger. In Lok Sabha, two Trinamool and one YSR Congress Party MPs face
proceedings. The Trinamool Congress wants to disqualify its two MPs (one of them is Sisir Adhikari, father
of Suvendu) for joining the BJP, and the YSRCP wants to disqualify its MP for “anti-party activities”.
56. Which of the following will replace [1] in the above passage?
(a) 10th (b) 9th (c) 8th (d) 7th
Head Office: 127, Zone II, MP Nagar, Bhopal |+91-7676564400| https://www.toprankers.com Page 13 of 36
58. Which of the following is the ground of disqualification?
(a) If an elected member voluntarily gives up his membership of a political party.
(b) If any independently elected member joins any political party.
(c) If any nominated member joins any political party after the expiry of six months.
(d) All of the above
59. Rules regarding merger were changed in which of the following amendments?
(a) 89th Amendment (b) 90th Amendment
st
(c) 91 Amendment (d) 97th Amendment
60. In which of the following landmark judgement of Supreme Court, the Constitutional validity of Anti-defection
law was upheld?
(a) Kihota Hollohon vs. Zachilhu (b) Indira Sawhney vs. Union of India
(c) Maneka Gandhi vs. Union of India (d) Golak Nath vs. State of Madras
Passage (Q.61-Q.65): India’s rank has slipped by six places from last year on the Henley Passport Index,
which lists the world’s most travel-friendly passports.
The Henley Passport Index comes at a time when countries are easing travel rules for international visitors
after almost two years since the onset of the Covid-19 pandemic.
The index ranks the passports of countries according to the number of destinations their holders can visit
without a prior visa. The rankings are based on the analysis of data provided by the International Air
Transport Association (IATA).
The firm’s Q4 Global Mobility report says, “The global mobility gap is at its widest point ever and continues
to expand due to proliferating barriers to entry erected since the outbreak of the pandemic. Many countries
in the global south have relaxed their borders in a concerted effort to revive their economies but there has
been very little reciprocity from countries in the global north, which have enforced some of the most
stringent inbound Covid-19-related travel restrictions. Even fully vaccinated travellers from countries at the
lower end of the Henley Passport Index remain locked out of most of the developed world.”
62. Which of the following is the correct chronological order of ranking from highest rank to least rank?
(a) USA, Belgium, New Zealand, Germany
(b) Germany, Belgium, USA, Australia.
(c) Belgium, USA, Australia, Germany.
(d) USA, Belgium, Germany, Australia.
64. India shares the rank with which of the following country?
(a) Tajikistan (b) Pakistan (c) Kyrgyzstan (d) None of the above
65. Which country has the weakest ranking in Henley passport index?
(a) Iraq (b) Afghanistan (c) Yemen (d) Morocco
Head Office: 127, Zone II, MP Nagar, Bhopal |+91-7676564400| https://www.toprankers.com Page 14 of 36
SECTION – C: LEGAL REASONING
Directions (Q.66 – Q.105): Read the comprehensions carefully and answer the questions based on it.
Passage (Q.66-Q.70): The Medical Termination of Pregnancy (Amendment) Bill, 2020 was introduced in
Lok Sabha on March 2, 2020 and passed on March 17, 2020. The Act specifies the grounds for terminating
a pregnancy and specifies the time limit for terminating a pregnancy. Under the Act a pregnancy may be
terminated up to 20 weeks by a married woman in the case of failure of contraceptive method or
device. The Bill allows unmarried women to also terminate a pregnancy for this reason. The requirement
for terminating pregnancy up to 20 weeks requires advice of one doctor. For 20-24 weeks advice of two
doctors is needed and for more than 24 weeks permission from medical board is needed in case of foetal
abnormalities. Lastly termination of pregnancy at any time can only be done if the life of pregnant woman
is at stake. The Medical Termination of Pregnancy Act, 1971, nowhere provides for the express or implied
consent of the husband or anyone other than the woman carrying the child.
Medical Boards: All state and union territory governments will constitute a Medical Board. The Board will
decide if a pregnancy may be terminated after 24 weeks due to substantial foetal abnormalities. Each
Board will have a gynecologist, pediatrician, radiologist/sonologist, and other members notified by the state
government.With regard to privacy a registered medical practitioner may only reveal the details of a woman
whose pregnancy has been terminated to a person authorized by law. Violation is punishable with
imprisonment up to a year, a fine, or both.
66. Seema Malhotra was medically examined and she was adamant to get the foetus aborted but the husband
refused. The husband refused to sign the papers giving his consent to terminate the pregnancy. Medical
Termination was conducted anyway. Husband filed a suit for the recovery of Rs. 30 lacs towards damages
on account of mental pain, agony and harassment against the wife. Based on the author's reasoning should
the damages be awarded to the husband?
(a) Damages should be granted since the husband is an integral entity in the future of child.
(b) Damages should be granted since the husband has undergone mental pain and agony due to loss of
his child.
(c) Damages should not to be granted since the husband has no locus standi to file the case.
(d) Damages should not to be granted since the Medical Termination of Pregnancy Act nowhere provides
for the express or implied consent of the husband.
67. The Medical Termination of Pregnancy (Amendment) Act, carves out an exception for carrying out
termination of pregnancy, to save the life of pregnant woman irrespective of length of pregnancy. Doctors
will not be unnecessarily prosecuted if they act in accordance with the rules in good faith. Dr, Madhu
terminated the pregnancy of Radhika, a minor girl, who is an alleged victim of rape as the continuation of
pregnancy would result in the loss of her life. The police have decided to prosecute Dr. Madhu. Decide.
(a) Prosecution shall fail since Dr. Madhu is a doctor of high stature.
(b) Prosecution shall succeed since Dr. Madhu did not take permission from her family.
(c) Prosecution shall succeed since Dr. Madhu acted in violation of the Medical Termination of Pregnancy
Act.
(d) Prosecution shall fail since Dr. Madhu acted in good faith so that he can safeguard her life and put her
out of danger.
Head Office: 127, Zone II, MP Nagar, Bhopal |+91-7676564400| https://www.toprankers.com Page 15 of 36
68. Parents should have the right to know of a child's pregnancy or abortion because they are ultimately
responsible for their child's care until she is eighteen. With the exception of a few circumstances, parents
are responsible for their children's mental and physical health until they are adults. A child should not be
allowed to make such a huge and potentially damaging decision about her health without the parent's
knowledge. Which of the following weakens the author's argument above as per the passage given.
(a) The parents deserve to know of their child's pregnancy because they will step up and deal with the
pregnancy more rationally than their child.
(b) Parents may react very negatively to a daughter's pregnancy and reject her at such a critical phase in
her life, adding to her mental stress and forcing her to terminate the pregnancy.
(c) If the child cannot relate to the parents enough to tell them there is most probably a reason, aside from
fear of disapproval for it.
(d) The girl regardless of age, is the only person who has the right to decide whether or not to continue
with the pregnancy.
69. Sarika has 2 sons and was pregnant with another child. Due to fetal abnormalities she decided to terminate
the 24 weeks pregnancy. The Medical board comprising of gynecologist, radiologist/sonologist, and other
members notified by the state government gave permission and the pregnancy was terminated. Was this
the correct procedure to follow as per the passage?
(a) Yes, as it is the prerogative of the Board to decide if a pregnancy may be terminated after 24 weeks
due to substantial fetal abnormalities.
(b) No, as Sarika was not a rape victim.
(c) No, as a pediatrician was not present in the board.
(d) No, because the consent of two doctors is needed to terminate pregnancy up to 24 weeks.
70. Bindu was a surrogate who agreed to carry the baby of John and Amy. When Bindu was 20 weeks
pregnant, the doctor discovered that the child will be born with down syndrome. Hearing this John and Amy
decided that they did not want the child anymore and asked Bindu to abort the child. When Bindu refused
to terminate the pregnancy, John and Amy went to court to make her abort the child. Can the court ask
Bindu to terminate the pregnancy as per the passage?
(a) Yes, because the child is of John and Amy.
(b) Yes, because the child has fetal abnormalities.
(c) No, because only the woman carrying the child has a say regarding termination of pregnancy.
(d) Yes, because the mother is the only one who has a say regarding termination of pregnancy.
Passage (Q.71-Q.74): Section 378 of IPC states “Any person with the intention to move or take a movable
property out of the possession of the property holder without his consent then this is called theft.” The
intention is an important ingredient to perform the crime of theft and it must be dishonest. The intention
must exist that the taker of the property intends to cause wrongful gain to one person or wrongful loss to
another person. It is not necessary that the taker of property must have wrongful gain it would be sufficient
if it causes wrongful loss to the property holder. If a person takes away any movable property without the
consent of the property holder even though temporarily and with an intention to return it later on, it would
amount to theft. Where a person takes another’s, property believing on the mistake of fact that he has the
right to take that property, he is not guilty of theft because there is no dishonest intention and mistake of
fact is excusable. The property which is permanently attached to the earth is known as immovable property
and which is not attached with earth and can move from one place to another is known as movable property
which is defined under section 22 of IPC. The subject matter of theft must be movable property it must not
be immovable property. Section 379 provides the punishment for theft. It states “Whoever commits theft
shall be punished with imprisonment of either description for a term which may extend to three years or
with a fine or with both.”
Head Office: 127, Zone II, MP Nagar, Bhopal |+91-7676564400| https://www.toprankers.com Page 16 of 36
71. Aman bought a piece of cloth worth Rs 1000/- from Xoco. He gave it for stitching to Bunny a tailor for
making a suit for him. After 3 days Aman went to collect his suit. Bunny demanded the stitching cost from
Aman but he refused and took away his suit without paying him.
(a) Aman is not guilty of theft as he was the owner of the suit.
(b) Aman is guilty of theft because he did not pay the tailor’s money.
(c) Aman is not guilty of theft because he had no dishonest intention.
(d) Aman is guilty because he took the suit from the tailor’s possession without paying his consent.
72. Sanay is Mr. Manoj’s secretary. One day while working in Mr. Manoj’s office Sanay removed a file in his
house which contained confidential information, made it available to an outsider and then returned it to the
office after two days. Decide.
(a) Sanay is not guilty because the file is still in the office.
(b) Sanay is guilty for theft because he took the file without Mr. Manoj’s consent.
(c) Samay is guilty because he had a dishonest intention.
(d) Samay is guilty because he moved the file out of Mr. Manoj’s possession without his consent with a
dishonest intention.
73. In a village called Zamby, Mohan took Zohan’s three cows against her consent and distributed them among
three people in his village. Decide.
(a) Mohan is not liable for theft because he was acting in good faith.
(b) Mohan is not guilty for theft because there is no wrongful gain of the thief.
(c) Mohan is guilty of theft because although there was no wrongful gain to the thief but there was wrongful
loss to the owner.
(d) Mohan is not guilty because the essentials of theft have not been fulfilled.
74. Z was going on a journey and therefore he gave his gold chain to A at that time. A had the possession of
that gold chain till Z returned. The following day, A sells that gold chain to B. Decide.
(a) A is liable for theft because the chain was Z’s.
(b) A is liable for theft because he sold the chain without Z’s permission.
(c) A is not guilty as he was entrusted with the chain.
(d) A is not guilty because the chain was not in Z’s possession so A has not committed theft.
Passage (Q.75-Q.79): Justices M. Duraiswamy and K. Muralishankar of the Madurai bench of the Madras
high court have held that there is a colossal difference between complaining that a person is acting against
the government and that person is protesting the policies of the government. The court was hearing a
petition filed by K Siva, a law graduate. He was not permitted to apply as a member of the Bar Council of
Tamil Nadu and Pondicherry. A police verification report against Siva had mentioned that 88 criminal cases
were registered against him between 2017 and 2019.
Most of these cases were lodged after the demonstrations held at Thoothukudi on May 22, 2018 demanding
closure of the Vedanta’s Sterlite copper plant as it had polluted the air and water and caused health hazards
to the people. Thirteen persons died during the protests, as police opened fire at them. The Madras high
court had on August 18, 2020 upheld the state government’s closure order. The Madurai bench has
reasoned that the duties of the state in protecting the environment are basically the rights of the people.
The bench relied on Article 51-A (g) of Part IV-A (Fundamental Duties), which deals with the duty to protect
and improve the natural environment and to have compassion for living creatures. The bench relied on the
Supreme Court’s observation that the mere fact that the petitioner was a part of a group and shouted
slogans cannot make him guilty of an offence, as the group was impressing upon the state the need to
follow the Directive Principles of State Policy enshrined in Article 47 of the constitution.
Head Office: 127, Zone II, MP Nagar, Bhopal |+91-7676564400| https://www.toprankers.com Page 17 of 36
75. Student A, troubled by the filthy and unhygienic conditions prevailing in his neighbourhood causing
widespread cases of dengue and chikungunya, due to the City Municipal Corporation’s apathy and
negligence, decides to organise a peaceful protest against the local government for their continued
inaction. Many persons join this protest, shouting slogans against the government for denying the locality
funds and medical assistance. Soon there is bloodshed and violence when the police start open-firing at
the protests. Several cases are filed against student A for motivating anti-government sentiments. Decide
student A’s liability?
(a) Student A should be held liable as he is solely responsible for organizing the protest which led to people
shouting slogans that were strongly against the government.
(b) Student A should not be held liable as he was too young and therefore incapable of understanding the
consequences of his actions.
(c) Student A should not be held liable as he was protesting for his rights and urging the government to
fulfil its own duty.
(d) Student A along with the rest of the protestors should be held liable for invoking violence against the
government and its machinery.
76. The Members of the Medical Welfare Association (MWA) with relation to the recent curbs on the import of
a vital drug for diabetes called Glipalamide, following ongoing trade restrictions by the state government to
increase domestic drug manufacturing capabilities leading to sudden shortfalls in the availability of this vital
medication, decide to assemble outside the Parliament in organized groups and demand the government
to resign. They encourage bystanders to join their protest to help them overthrow the government. Cases
are filed against the MWA for their seditious acts. Decide The MWA’s liability?
(a) The MWA is not liable as they are merely protesting government policy which is responsible for causing
a huge shortfall in vital medication.
(b) The MWA is not liable as there is huge difference between protesting against the government and
protesting for collective interest.
(c) The MWA should be held liable for purposely misleading innocent by-standers to protest for a cause
they don’t personally believe in.
(d) None of the above.
77. Company XYZ located on the outskirts of the village Dholakpur, is allegedly dumping its chemical waste
into a stream which is the main source of water for most of the villagers. Dholu, a law graduate from The
Dholakpur Law University decides to file a case against the company for causing environmental damage
and posing health hazards to the villagers. Dholu, frequently shouts slogans and organizes protests outside
the Tehsildar’s office to compel the Sarpanch to order the suspension of the Company’s functioning, but in
vain. He had recently applied for a government job at The Law and Research Centre in Dholakpur, he was
denied the job on the grounds that he had a criminal background. Adding to his misery the police too had
filed a charge-sheet against him for criminal conspiracy against the Sarpanch. Decide if Dholu is eligible to
get the job as per the passage.
(a) Dholu is eligible for the job as protesting for a righteous cause furthered by article 52-A (g) of Part IV-
(A) (Fundamental Duties) of the Constitution doesn’t account for a criminal act.
(b) Dholu is eligible for the job as he shouldn’t be held guilty for an act which isn’t an offense and is meant
to encourage public discourse about citizen rights and urge the government to recognise its own duties
toward its people.
(c) Dholu isn’t eligible for the job as he is notoriously infamous for his actions against the government.
(d) Dholu can be eligible for the job in the future if he stops protesting against the government and starts
protesting against the governance.
Head Office: 127, Zone II, MP Nagar, Bhopal |+91-7676564400| https://www.toprankers.com Page 18 of 36
78. Kumar Fahid, a prominent environmental activist known for being critical of many government policies,
started his own show called the Green-Time TV, this show was meant to mirror the extent of the country’s
environmental loss and measures taken to allay further ecological concerns. It also featured daily interviews
with youth environmental activists who were keen on raising awareness on government policies that were
harming indigenous people and tribal groups because of the rapid urbanisation of several swathes of forest
land. Popular Journalist and Media person, AmitavGhoswami files a case in a district court against Kumar
Fahid, for his misinformation campaign and calls the show anti-national. The Court then held that the
remaining episodes of the Green-Time TV should not be allowed to be broadcast as they are against public
interest. Is the court correct in its interpretation?
(a) Yes, as the facts of the case make it clear that airing inflammatory content against the government is
useless and that calls for strict action.
(b) Yes, as the content of the show went against the government and its policies, therefore damaging and
distorting the general public opinion.
(c) No, it is nowhere clear, going by the facts of the case that the show was against the government.
(d) Yes, as there was a hidden agenda for Kumar Fahid, known for criticising the government.
79. According to the author of the passage, which of the following would not be in contravention to the
Fundamental Duty mentioned in the above passage?
(a) Construction of roads and lanes on vulnerable mountainous terrain to connect cities to
rural/underdeveloped areas.
(b) Allowing chemical effluents to pollute rivers that flow in uninhabited areas.
(c) Creating inhumane conditions for slum-dwellers by dumping the city’s waste in slum localities.
(d) Producing fossil fuel and mining large amounts of coal that in turn aid climate change and increase the
global climate temperature.
Passage (Q.80-Q.85): Two adults have a right to choose their matrimonial partner irrespective of their
religion. The Allahabad High Court has reaffirmed while granting protection to an inter-faith couple. A
Division Bench granted protection to the inter-faith couple and said that not even their parents could object
to their relationship.
"It cannot be disputed that two adults have right of choice of their matrimonial partner irrespective of religion
professed by them...As the present petition is a joint petition by the two individuals who claim to be in love
with each other and are major, therefore, in our considered opinion, nobody, not even their parents, could
object to their relationship."
On the said application, the District Magistrate had called for a report from the concerned police station. As
per the report, the boy's father is not agreeable to the marriage though his mother is ready for the same.
Both parents of the girl are opposed to the marriage.
In view of the same, the couple approached the High Court, claiming that there was a threat to their lives.
The Court took note of the fact that both the petitioners are major aged 19 and 24 years.
The Court proceeded to grant them protection while clarifying that the order is not any final opinion
regarding the age of the petitioners as the findings are only prima facie in nature for the purposes of
deciding the issue regarding protection of the life of the petitioners.
Source: https://www.barandbench.com/news/litigation/adults-have-right-to-choose-their-matrimonial-
partner-irrespective-of-religion-allahabad-high-court
80. Rama and Shruti are aged 18 and 19, respectively, and have been living in the same village all their lives.
After having been friends for over 14 years, they finally decided to get married as they were in love with
each other. As both their families were cordial to each other and were of a respected caste, they assumed
that their decision would be welcomed. To their dismay, neither of their parents was amenable to the union
and denied permission outrightly. Can they still get married?
(a) Yes, as they are of consenting age and of the same caste.
(b) No, as they are not adults yet and cannot get married.
(c) No, as their guardians have not yet bestowed upon them the power to consent for the same.
(d) Yes, as the SC has cleared the position of law on the same by stating that if the individuals are major,
not even the parents can object to the union.
Head Office: 127, Zone II, MP Nagar, Bhopal |+91-7676564400| https://www.toprankers.com Page 19 of 36
81. In another matter of inter-caste marriage Julie, a Christian, and Hamid, an atheist, the petitioners were
seeking the protection of the SC insofar as protecting their right to marry as an inter-caste, inter-faith couple.
It was clear that their parents did not consent to the same, and thus, the matter was under consideration.
Can they be afforded protection?
(a) No, as Hamid does not belong to any faith and cannot be considered to be an interfaith couple.
(b) Yes, as they are clearly people of two different faith, and their rights are safeguarded.
(c) No, as the constitution only envisages the right to religion insofar as worship and practice are
considered.
(d) None of the above.
82. Sudha and Jojo, a homosexual couple, had run away from their home wanting to marry each other, being
of consenting age and of the same faith. However, before they could appear before the registrar of
marriages the next day, they were arrested by the police on account of the complaint filed by their parents.
They were denied marriage on account of the same, and they preferred a petition as against the High Court
of Delhi. Is their action tenable as per the passage?
(a) No, as homosexual marriages are not legal in India.
(b) Yes, as they are 2 individuals who are adults.
(c) Yes, as their parents do not get to have a say in the matter.
(d) No, as inter-faith marriages are only legal within inter-gender marriages.
83. Kalyan and Saloni were two underage kids, of 15 and 17 years old, and were madly in love with each other.
This motivated them to marry each other, thus they decided to elope. However, they were caught by their
relatives before they could get married. When the matter was brought infront of the court,the court refused
to grant them protection to live together. Is the decision of the Court correct?
(a) It is correct as they were not adults.
(b) It is incorrect since the same was not warranted before pleading for the same.
(c) It is unclear since the nature of a marriage may be dubious.
(d) It is incorrect as the parents cannot object to the relationship.
84. Under which article/provision would an aggrieved inter-faith couple of consenting age approach the Courts
for granting them protection in marrying each other?
(a) Article 226 of the Constitution of India.
(b) Article 32 of the Constitution of India.
(c) Article 136 of the Constitution of India.
(d) Cannot be determined from the passage.
85. Veeru and Gopa were caught trying to marry each other in the registrar's office by their parents and were
beaten there by them. Upon bringing an action against their parents and relatives for brutality and
discrimination, the SC of India had granted them the leave to marry and were compensated. However, a
parallel claim was filed in the lower courts against the registrar for allowing them to marry even though the
dissent of the parents was apparent in front of him. Can the action sustain?
(a) No, as the registrar is not personally liable for every marriage.
(b) No, as the registrar bears no liability arising out of the marriages he registers.
(c) No, as the dissent of the parents is not a valid contention for recourse.
(d) No, as the parents and relatives themselves took the law into their own hands, breaching public order.
Head Office: 127, Zone II, MP Nagar, Bhopal |+91-7676564400| https://www.toprankers.com Page 20 of 36
Passage (Q.86-Q.90): The concept of contempt of Court is several centuries old. In England, it is a
common law principle that seeks to protect the judicial power of the king, initially exercised by himself, and
later by a panel of judges who acted in his name. Over time, any kind of disobedience to judges, or
obstruction of the implementation of their directives, or comments and actions that showed disrespect
towards them came to be punishable. The law codifying contempt classifies it as civil and criminal. Civil
contempt is fairly simple. It is committed when someone willfully disobeys a Court order, or willfully
breaches an undertaking given to Court. Criminal contempt is more complex. It consists of three forms: (a)
words, written or spoken, signs and actions that “scandalize” or “tend to scandalize” or “lower” or “tends to
lower” the authority of any Court (b) prejudices or interferes with any judicial proceeding and (c) interferes
with or obstructs the administration of justice. Making allegations against the judiciary or individual judges,
attributing motives to judgments and judicial functioning and any scurrilous attack on the conduct of judges
are normally considered matters that scandalise the judiciary. The rationale for this provision is that Courts
must be protected from tendentious attacks that lower its authority, defame its public image and make the
public lose faith in its impartiality.
The punishment for contempt of Court is simple imprisonment for a term up to six months and/or a fine of
up to ₹. 2,000.
Fair and accurate reporting of judicial proceedings will not amount to contempt of Court. Nor is any fair
criticism on the merits of a judicial order after a case is heard and disposed of.
86. Mr. Mohit, who is a noted lawyer and activist tweeted that “The CJI is enjoying his ride on a motorbike
worth INR 40 lakh belonging to a political leader, at a time when he has kept the Supreme Court in lockdown
mode denying citizens their fundamental rights to access justice”. Decide whether he is liable for contempt
of Court
(a) Yes, he is liable because he has voluntarily maligned CJI.
(b) No, he is not liable because his statement will be protected as Fundamental Right of Speech and
expression.
(c) He is not liable because his remark stated the truth.
(d) He is not liable as his tweet was just his opinion without any malice.
87. Mr. A is a public servant who was handling the work of cleaning and sanitizing Delhi Metro premise. It was
later found that he was using low standard sanitizers for which a suit was filed against him in Delhi. Judge
found him guilty and imposed a fine of fifty thousand rupees. Mr. A was not satisfied with the order and he
made a tweet that “Justice is denied to me and it is due to the incompetence of the judge because he had
personal interest in the matter and knows why he ruled against A I will file an appeal.” Decide whether Mr.
A is liable for contempt of Court.
(a) Yes, because his remarks made against the Judge are scandalous and can prevent the course of
justice.
(b) No, he was using his freedom of speech and expression.
(c) No, because he simply stated his opinion.
(d) Yes, because he said that the judge didn’t understand anything at all.
88. Mr. Shashank is a young lawyer who is practicing in Delhi High Court. He filed a PIL in Delhi HC for removal
of large hoardings of politicians in the vicinity of Court. But, the Court rejected his PIL without finding any
merits in it. It infuriated Mr. Shashank filed an appeal where he claimed that the Delhi HC is unable to
understand his grievance and therefore justice was not done to him. Later, one of the judges of Delhi HC
initiated a contempt proceeding against him. Decide whether he is liable for contempt of Court.
(a) Yes, because his remarks made against the judge are scandalous and can prevent the course of
justice.
(b) No, because any citizen can comment or criticize the judgement of this Court and appeal against it on
merits.
(c) No, because he only said justice was not done to him.
(d) Yes, as he stated that the judiciary failed to address his grievances.
Head Office: 127, Zone II, MP Nagar, Bhopal |+91-7676564400| https://www.toprankers.com Page 21 of 36
89. Mr. Xora, a practicing lawyer, writes a blog on a website criticizing a judgement passed by the Allahabad
High Court for failing to follow the correct position of law settled by the Supreme Court in an earlier decision.
Whether the Allahabad HC can publish Mr. Xora for committing criminal contempt?
(a) Yes, because his remarks made against the Judge are scandalous and can prevent the course of
justice.
(b) No, because innocent publication, distribution of matter and reasonable and fair criticism of judicial act
is permissible.
(c) No, as fair criticism on the merits of a judicial order after a case is heard and disposed of is not contempt
of Court.
(d) No, as fair criticism on the merits of a judicial order while a case is heard and disposed of is not contempt
of Court.
90. During the prevalence of Coronavirus pandemic, the Allahabad HC passed an order, in a Public Interest
Litigation, stating that no eviction or demolition can be done by any person/authority during this period. Mr.
X was shown a copy of this order by residents of a gated residential society when he went there to demolish
the society gate. Mr. X tore down the copy of the order handed over to him stating that “High Court Judges
have got no work to do except passing whimsical orders while sitting in air-conditioned rooms and
demolished the gate of the society. Whether Mr. X is liable to be punished for criminal contempt?
(a) Yes, because his statements interfere with, the due course of any judicial proceeding as he is willfully
breaching order of the Court.
(b) No, he was using his freedom of speech and expression.
(c) No, as this amounts to fair criticism on merits of judicial order;
(d) Yes, because in COVID pandemic he was demolishing the building obstructing the course of justice.
Passage (Q.91-Q.95): The NDPS Act, 1985 is the principal legislation applicable in India, through which
the state regulates the operations of narcotic drugs and psychotropic substances. It provides a stringent
framework for punishing offences related to illicit trafficking in narcotic drugs and psychotropic substances
through imprisonments and forfeiture of property. Section 27A of the NDPS Act, 1985, prescribes the
punishment for financing illicit traffic and harbouring offenders. The NDPS Act says that “narcotic drug”
means “coca leaf, cannabis (hemp), opium, popy straw and includes all manufactured drugs”. Further,
“psychotropic substance” refers to “any substance, natural or synthetic, or any natural material or any salt
or preparation of such substance or material included in the list of psychotropic substances specified in the
Schedule.” The aim of the NDPS Act is to prohibit “the manufacture, production, trade, use, etc. of narcotic
drugs and psychotropic substances”, except for medical or scientific purposes by medical experts.
The Act provides for lawmakers to expand the list of psychotropic substances or remove items from it on
the basis, of among other things, “information and evidence which has become available to it with respect
to the nature and effects of, and the abuse or the scope for abuse of, any substance (natural or synthetic)
or natural material or any salt or preparation of such substance or material”.
In Sami v. State of Uttar Pradesh, the police caught a person with 25 kilograms of unlicensed Bhang (a
cannabis preparation) on him. It was held that while the Act defines cannabis(hemp), there is no explicit
inclusion of bhang under this definition and that it was not a narcotic or psychotropic substance. Thus, its
possession would not lead to a conviction under the NDPS Act if procurement of bhang is within permissible
limits of 25 kgs.
91. X and Y are US citizens of 25 and 26 years old respectively. They knew that consumption of drugs is illegal
in their country so they came India to consume the same. They tried Indian Drugs in very little amount in
order to understand its quality and nature of substances. But while tasting a very less amount of drugs
without a doctor’s prescription, both get caught by police officials. Now decide whether they can be held
liable for consumption of drugs taking the objective of the act in consideration?
(a) No, both of them cannot be held liable as they both are US citizens over whom Indian laws are not
applicable.
(b) No, both of them cannot be held liable as NDPS act allows for less consumption of drugs up till 25 kgs
therefore minimal amount of consumption is not illegal.
Head Office: 127, Zone II, MP Nagar, Bhopal |+91-7676564400| https://www.toprankers.com Page 22 of 36
(c) Yes, both of them can be held liable as NDPS act prohibits of consumption of drugs therefore even a
minimal amount of consumption is illegal.
(d) No, because they are majors, they have the right to decide.
92. Cran and Berry are UK citizens of 32 and 28. They knew that consumption of drugs is illegal in their country
so they came India to consume the same. They tried Indian Drugs - Bhang in very little amount (5kg) in
order to understand its quality and nature of substances. But while tasting a minimal number of drugs
without a doctor’s prescription, both get caught by police officials. Now decide whether they can be held
liable for consumption of bhang?
(a) No, both of them cannot be held liable as they both are UK citizens over whom Indian laws are not
applicable.
(b) No, both of them cannot be held liable as NDPS act is not applicable on Bhang in less amount hence
consumption is not illegal.
(c) Yes, as bhang is a narcotic substance and the NDPS Act categorically bans consumption of the same.
(d) Yes, asmanufacture, production, trade and use of narcotic drug and psychotropic drug except medical
and scientific purposes is punishable .
93. In Sami v. State of Uttar Pradesh as in the passage, the police caught a person with 25 kilograms of
unlicensed Bhang (a cannabis preparation) on him. It was held that while the Act defines cannabis(hemp),
there is no explicit inclusion of bhang under this definition and that it was not a narcotic or psychotropic
substance. Further if it was held that , its possession would not lead to a conviction under the NDPS Act.
However, if bhang contained Tetrahydrocannabinol (THC) then it would come under narcotic and
psychotropic substance. Suresh is a law student and was also a chain smoker. He was famously known
for his psychotropic substances consuming habits therefore he was renamed as ‘ACID’ among his friends.
But one day when Suresh was about to deliver some of the substances i.e.charas and bhang (with THC),
he fell inside a police trap and was substantially charged under NDPS act. Decide can ACID be held liable
for the same under the NDPS act?
(a) No because ACID was not himself consuming these substances, he was only delivering the same work
as a peddler.
(b) No because Bhang is not covered under NDPS act.
(c) Yes, because consumption, possessionand distribution of psychotropic substances without a doctor’s
prescription, a person can be booked.
(d) Yes, only and if bhang was more than 25 kgs.
94. Section 54 of NDPS act states “54. Presumption from possession of illicit articles. In trials under this Act, it
may be presumed, unless and until the contrary is proved, that the accused has committed an offence
under this Act in respect of: (a) any narcotic drug or psychotropic substance or controlled substance;
(b) any opium poppy, cannabis plant or coca plant growing on any land which he has cultivated; (c) any
apparatus specially designed or any group of utensils specially adopted for the manufacture of any narcotic
drug or psychotropic substance or controlled substance; or (d) any materials which have undergone any
process towards the manufacture of a narcotic drug or psychotropic substance or controlled substance.”
X was found to possess opium consequently he was accused of possession and consumption of drugs
under the NDPS act. In district Court, the accused was asked to produce contrary evidence that means the
burden of proof was put on the accused. Accused filed a writ petition against wrongful proceedings taken
in district Court where he raised ‘onus of burden proof shall be on the petitioner, not on the accused.’
Decide what view can be taken by the High Court?
(a) HC will dismiss the petition in favor of the accused as it is the general principle of law i.e., the onus of
burden of proof shall lie on petitioner only.
(b) HC will dismiss the petition against the accused as in accordance with section 54 of the NDPS act, the
accused is obliged to provide evidence against the petitioner.
(c) Accused cannot file a writ petition as a person who seeks equity must come with clean hands.
(d) None of the above.
Head Office: 127, Zone II, MP Nagar, Bhopal |+91-7676564400| https://www.toprankers.com Page 23 of 36
95. Maresh is a medical student and was also a chain smoker. He was famously known for his psychotropic
substances consumption therefore he was renamed as ‘MD’ among his friends. However, one day he was
found growing coco leaf in his room. The police arrested him instantly. But Maresh insisted that it was for
his medical science project. Decide can MD be held liable for the same under the NDPS act?
(a) Maresh is guilty asproduction of narcotic drugs is prohibited except by medical experts.
(b) Maresh is not guilty as growing coco leaf was a part of his project.
(c) Maresh is guilty as he was found manufacturing coco leaf by growing it in his room.
(d) Maresh is not guilty as his research might lead to a breakthrough in the field of medical science.
Passage (Q.96-Q.100): The Supreme Court observed that, in a consumer case, the onus of proof that
there was deficiency in service employed is on the complainant and without any proof of deficiency, the
opposite party cannot be held responsible for deficiency in service, Justices Hemant Gupta and V.
Ramasubramanian observed.
If the complainant is able to discharge its initial onus, the burden would then shift to the respondent in the
complaint. The rule of evidence before the civil proceedings is that the onus would lie on the person who
would fail if no evidence is led by the other side. Therefore, the initial burden of proof of deficiency in service
employed was on the complainant, but having failed to prove that the result of the sample retained by the
appellant at the time of consignment was materially different than what was certified by the appellant, the
burden of proof would not shift on the appellant. Thus, the Commission has erred in law to draw adverse
inference against the appellant.
In this regard, the bench referred to the decision in Ravneet Singh Bagga v. KLM Royal Dutch
Airlines (2000) 1 SCC 66 in which it was observed thus: The deficiency in service cannot be alleged without
attributing fault, imperfection, shortcoming or inadequacy in the quality, nature and manner of performance
which is required to be performed by a person in pursuance of a contract in relation to any service
employed. The burden of proving the deficiency in service employed is upon the person who alleges it.
96. Mr. Ghasitaram was witnessing berthing difficulties, cough and fever therefore his brother Mr. Haldiram
admitted him to KGF hospital, and on primary diagnosis doctors revealed that the patient was suffering
from covid 19 as well as his preexisting chronic lung dieses ILD and was already in a critical condition.
Doctors isolated the patient and treated him according to the Covid guidelines of ICMR despite all that he
died. Haldiram claimed that due to the deficiency of service on the part of KGF doctors his brother died.
Decide the Hospital’s liability:
(a) KGF hospital is liable as there was a deficiency of service on their part and due to this Mr. Ghasitaram
died.
(b) KGF hospital is not liable as they did not cause Ghasitaram’s death, he died due to his medical
condition.
(c) Neither KGF Hospital nor Mr. Haldiram will be liable for there is no deficiency of services on either of
the parties side.
(d) KGF hospital is not liable as they did no fault or imperfection on their part that could have led to
Haldiram’s death.
97. Mr. Vex was fond of different types of candles and used to lit them in the evening every day throughout his
house. One day while he was enjoying his evening with a coffee in candle lights, his neighbors called him
for small talk and when he returned, his house was on fire, suddenly a passer-by Mr. Water came who was
afire brigade by a profession came to Mr. Vex’shelp tried to extinguish the fire however due to his fault fire
got worse and Mr. Vex’s house burned to ashes. Mr. vex filed a complaint against that fire brigade claiming
deficiency in service.
(a) Mr. Water is liable for deficiency of service on his part as due to his fault the fire got worse and Mr.
Vex’s house burned to ashes.
(b) Mr. Water is not liable as all the incidence was a mere accident.
(c) Mr. Water is not liable as Mr. Vexdid not employ his services.
(d) Mr. Water is liable for it was his fault, imperfection, shortcoming in providing services which led to Mr.
Vex’s house on fire
Head Office: 127, Zone II, MP Nagar, Bhopal |+91-7676564400| https://www.toprankers.com Page 24 of 36
98. Mr. Baburao Ganpatrao Apte is a cynical man and trusts no one in this world. He had a very old model of
washing machine of Octopus Company of which the company itself has stopped the production years ago.
One day the washing machine stopped working therefore he called Mr. Raju the service man of Octopus
Company to repair it. Mr. Raju repaired it but advised Mr. Babu Rao Apte to buy new model and left. Babu
Rao Apte was not satisfied with his work and claimed deficiency of service from Raju. Which of the following
statement is true?
(a) The initial onus of prove that there was no deficiency of service is on Mr. Raju.
(b) The onus of proof that there was deficiency of service is on neither of the parties; .
(c) Mr. Baburao Ganpatrao Apte could not claim compensation without providing any material evidence
and proof of Mr. Raju’s deficiency of service.
(d) The onus of proof that there was deficiency of service is on Octopus Company.
99. Whale Ltd. is engaged in business to Dolphin Ltd. for years, for providing services for inspection of
groundnut procured by it for the purpose of exporting the same to Greece and Other lands. However, on a
field inspection of consignment Dolphin Ltd. found that groundnut did not meet the products specifications
at the time of loading of consignment. Dolphin limited filed a complaint in the National Consumer Disputes
Redressal Forum. Decide.
(a) Whale Ltd. is liable for deficiency of service as the groundnut inspected by it did not meet the products
specifications.
(b) Whale Ltd. is not liable as they did no fault or imperfection on his part.
(c) Whale Ltd. is not liable as he is not contractually bound to meet the product specification and this was
only a mere formality.
(d) Whale Ltd. is liable for there was a fault on their part
100. Which of the following statement is not true in reference to the given passage?
(a) The burden of proof is always on the person who claims that there was a deficiency of service on the
part of the service provider.
(b) If sample retained by the appellant at the time of consignment is materially different than what was
certified by the appellant, the burden of proof would shift on the appellant.
(c) A service provider is needed to act in a manner that a reasonable and prudent service provider would.
(d) Without any proof of deficiency, the opposite party cannot be held responsible for deficiency in service.
Passage (Q.101-Q.105): In the wake of the prevailing situation in Afghanistan where the Taliban have
seized power, India has introduced a new category of e-visa for Afghan nationals to fast-track their
applications for entry into India. These visas will be valid for six months only and will be granted only after
security clearance, sources said. “Ministry of Home Affairs (“MHA”) reviews visa provisions in view of the
current situation in Afghanistan. A new category of electronic visa called “e-Emergency X-Misc Visa”
introduced to fast-track visa applications for entry into India,” a statement from MHA.
Sources said the visa is being granted to facilitate the stay of Afghan nationals fleeing Afghanistan for six
months in India. There is no clarity yet on what will happen after this validity period expires. India does not
have a refugee policy and grants shelter to foreigners facing persecution in their countries on a case-to-
case basis.
“Such a visa may be granted only with single entry and for the specific duration taking into account the
purpose of visit. If the visa is granted for a period of stay exceeding 185 days. This visa will be non-
extendable and non-convertible to any other type of visa,” the MHA policy document on X-Misc visa says.
India has in the past granted long duration visas to Afghan nationals — of all religions — facing persecution
in that country and a large number of Afghans who fled the country during the first Taliban takeover. Other
foreigners in this category include those from Pakistan, Iraq, Sudan, foreigners of Pakistani origin and
stateless persons.
Head Office: 127, Zone II, MP Nagar, Bhopal |+91-7676564400| https://www.toprankers.com Page 25 of 36
101. Aaban Mohamad and Shahid were companion of each other and fled away form Afghanistan on 26th of
March and reached India on 28th of March and are now refugees and have obtained visa under e-
Emergency X-Misc Visa for 6 months. But they stayed in Jammu till September for complete 6 months, and
have not left Indian frontiers since then. Decide.
(a) They have completed the Time period given, so they have to go through punishments.
(b) No, they can’t be punished as they are refuges and they are here to shorten their problem not to
escalate.
(c) Their visa shall now be non-extendable and non-convertible to any other type of visa.
(d) None of the above.
102. Sundar Pichai an employee at Google in Silicon Village is an Indian Origin Citizen but is resident of United
Kingdom. He used to come to India often to meet his parents. He wanted to visit India from UK and
consequentlyapplied for visa under E-Visa under the e-Emergency X-Misc for 6 months. Decide
(a) His visa will be rejected for the visa is for Afghan nationals who are refugees and fleeing from their
home country;
(b) His visa will be accepted for it is granted for entry into India;
(c) His visa will be rejected for the visa is granted for single entry to India only;
(d) His visa will be accepted for he has applied for the visa for 6 months as per the provision.
103. Sharbat Gula and Ashraf Ghani are refugees and migrants from Afghanistan and Bangladesh as they were
facing persecution in their home country they fled to India, wherein Sharbat Gula has procured visa e
emergency X Misc Visa, however as India does not have any Refugee policy or have not signed any
convention regarding it with Bangladesh, Ashraf Gula has not procured any visa. In the light of the present
facts decide Indian Government declare can Ashraf Ghani an illegal Migrant/refugees?
(a) Yes, Ashraf Ghani is an Afghan citizen and he can be declared Illegal migrant/refugee by the competent
Authority.
(b) Yes, Ashraf Ghani can be declared illegal migrant/refugee by the Competent Authority.
(c) Can’t be determined as this needs to decided case to case basis.
(d) No, Ashraf Ghani can’t to declared illegal Migrant/Refugee as, he lives under, FRRO and can’t be
declared Illegal as its inhuman and Constitution does not allow us to do that for it ensures right to live
with dignity.
104. Due to Taliban attack on Afghanistan’s administration in its capital, Many Afghan citizens have left
Afghanistan, Ram Verma was one of them. He forged a fake passport and have criminal charges against
him. Considering India is an active member of United Nations High Commissioner for Refugees, what will
happen?
(a) Ram Verma will not be able to enter India as he has forged the passport.
(b) Ram Verma must have done crimes in Afghanistan including forging passport but Taliban is bigger
threat than anything, so he should be allowed to enter India.
(c) Ram Verma is not a potential threat to the country, but have criminal charges.
(d) Ram will not be allowed to enter India for he does not have the requisite visa.
105. Government of India has started implementing Citizen Amendment Act (“CAA”) in India despite the
enforceability of the Foreigners Act, 1946. .The Citizen Amendment Act strictly refuses Muslims to be the
citizen of country. Shayra, an engineer from profession and Muslim by religion, when Taliban attacked
Afghan and gained control over government she fled from Afghan to India. Before CAA came into its
applicability, she had procured emergency visa for 6 months owing to state of affairs in Afghan. Decide. .
Decide.
(a) Shayra will be allowed to stay in India for the law accords equal protection to all.
(b) Shayra will not be allowed to stay in India as there is clash between the provisions of CAA and category
of visa launched by India, hence can not be determined.
(c) Shayra will be allowed to stay in India for she has procured the requisite visa;
(d) Shayra will not be allowed to stay in India in India as she is a Muslim and CAA bars the same.
Head Office: 127, Zone II, MP Nagar, Bhopal |+91-7676564400| https://www.toprankers.com Page 26 of 36
SECTION - D: LOGICAL REASONING
Directions (Q.106-Q.135): Read the passage carefully and answer the questions that follow.
Passage (Q.106-Q.108): There is a growing feeling, among those who have the responsibility of managing
large economies, that the discipline of economics is no longer fit for purpose. It is beginning to look like a
science designed to solve problems that no longer exist. A good example is the obsession with inflation.
Economists still teach their students that the primary economic role of government - many would insist, its
only really proper economic role - is to guarantee price stability. We must be constantly vigilant over the
dangers of inflation. For governments to simply print money is, therefore, inherently sinful. If, however,
inflation is kept at bay through the coordinated action of government and central bankers, the market should
find its "natural rate of unemployment," and investors, taking advantage of clear price signals, should be
able to ensure healthy growth. These assumptions came with the monetarism of the 1980s, the idea that
government should restrict itself to managing the money supply, and by the 1990s had come to be accepted
as such elementary common sense that pretty much all political debate had to set out from a ritual
acknowledgment of the perils of government spending.
This continues to be the case, despite the fact that, since the 2008 recession, central banks have been
printing money frantically in an attempt to create inflation and compel the rich to do something useful with
their money, and have been largely unsuccessful in both endeavours. We now live in a different economic
universe than we did before the crash. Yet the language of public debate, and the wisdom conveyed in
economic text books, remain almost entirely unchanged.
One expects a certain institutional lag. Mainstream economists nowadays might not be particularly good at
predicting financial crashes, facilitating general prosperity, or coming up with models for preventing climate
change, but when it comes to establishing themselves in positions of intellectual authority, unaffected by
such failings, their success is unparalleled. To this day, economics continues to be taught not as a story of
arguments but rather as something more like physics, the gradual realization of universal, unimpeachable
mathematical truths.
106. Which of the following most accurately expresses the main point of the second paragraph of the passage?
(a) The 2008 financial crisis has affected our financial systems so severely that we cannot do anything to
make it right.
(b) The 2008 financial crisis has changed many concepts which we assumed and, therefore, we must
evolve in our understanding of economics.
(c) The 2008 financial crisis made us print a lot of money in an attempt to force the rich to use their money,
but it did not work.
(d) The 2008 financial crisis should be taught in our textbooks as a reminder to what we could have done
differently.
108. Which of the following is the author most likely to agree with?
(a) Governments cannot print excess money as it would lead to inflation.
(b) Today, economists are successfully predicting financial crashes and preventing climate change.
(c) Banks have printed extra money in the past to induce inflation in the markets artificially.
(d) Economists are moving towards the mathematical aspect of economics as it provides more promising
results.
Head Office: 127, Zone II, MP Nagar, Bhopal |+91-7676564400| https://www.toprankers.com Page 27 of 36
Passage (Q.109-Q.113): The deliberate or unintentional spread of misinformation, despite capturing
widespread public attention, remains as rampant as ever, showing up recently in the form of false claims
about COVID-19 vaccines, the Capitol riot, and many other instances. This “infodemic” is polarizing politics,
endangering communities, weakening institutions, and leaving people unsure what to believe or whom to
trust. It threatens the foundations of democratic governance, social cohesion, national security, and public
health.
Misinformation is a long-standing problem that demands long-term, sustainable solutions as well as short-
term interventions. We've seen a number of quicker, technological fixes that improve the social media
platforms that supply information. Companies like Facebook and Twitter, for example, have adjusted their
algorithms or called out problematic content. We've also seen slower, human-centred approaches that
make people smarter about the media they demand to access online. Evidence-driven educational
programs, for instance, have made people better at discerning the reliability of information sources,
distinguishing facts from opinions, resisting emotional manipulation, and being good digital citizens.
It hasn't been enough. If we're to stop misinformation and its insidious effects, we need to radically expand
and accelerate our counterattacks. It will take all sectors of society: business, non-profits, advocacy
organizations, philanthropists, researchers, governments, and more. We also need to balance our efforts.
For too long, too many resources and debates have focused on changing the technology, not educating
people. This emphasis on the supply side of the problem without a similar investment in the demand side
may be a less effective use of time and energy and clearly, it has been the opposite of the ideal steps that
needed to be taken. While technology-centered, self-policing solutions—filtering software, artificial
intelligence, modified algorithms, and content labeling—do have the ability to make changes quickly and
at scale, they face significant ethical, financial, logistical, and legal constraints.
109. Which of the following statements will the author deem true?
(a) The spread of misinformation is not a recent problem that has cropped up in the world.
(b) Amidst the huge amount of negative news that’s emanating in the society today some positive
misinformation is good for us as well, especially during the pandemic times.
(c) Institutions that supply information can themselves solve the problems with certain steps. Only the
intent from these institutions is needed. It has been lacking amongst them. Their steps haven’t been
enough.
(d) None of the above.
110. Which of the following, if true, weakens the author's claim?
(a) A startling research observation has been that 70% of the information available on the internet today is
misinformation.
(b) It has been observed by a large number of studies that major corporations distribute more information
than individuals.
(c) International laws do not prohibit spreading misinformation by citizens of another nation.
(d) All people are against spreading and receiving misinformation, yet they are forced to get it because of
careless policies of organizations supplying them with such false facts.
111. Consider the following instances –
1. Tony Stark receives a message from Steve Rogers (a prankster, by the way) that the earth is a flat
surface and the people who call it round are mentally unstable. There are also some supposedly "true
scientific facts" to support this claim. Tony, in his innocence, believes this and forwards the message
unknowingly.
2. Albert is a pedant and a researcher who devours knowledge. He gets fascinated by Facebook and
Twitter. After some time, though, he gets frustrated with misinformation and quits social media
altogether.
3. The new sound space social media Addabaaz has become a field-space for misinformation with the
celebrity Angana Raha, spreading misinformation and hate. Addahouse changes their policy that
whenever such instances occur, and they receive complaints from the users, they shall remove the
problematic content.
Which of the above instances can be said to be examples of what has been mentioned in the passage?
(a) Both 1 and 2 (b) Both 2 and 3
(c) Both 1 and 3 (d) All 1, 2 and 3
Head Office: 127, Zone II, MP Nagar, Bhopal |+91-7676564400| https://www.toprankers.com Page 28 of 36
112. Who should have the most significant responsibility in taking steps to immediately stop the menace of
misinformation out of the following?
(a) The ordinary people, because they are the victims of the misinformation.
(b) The companies, because they are technologically much more advanced than the means available to
counter the spread of misinformation, plus the misinformation is happening on their platforms, so they
are in a better position to stop this.
(c) The political class, because they can educate the common folks in a much better way than anyone
else.
(d) All of the above have equal responsibility.
113. The statement "Clampdown on information providing organizations is not enough" is:
(a) Probably True (b) Definitely True
(c) Probably False (d) Definitely False
Passage (Q.114-Q.118): A lot of us think that the World Wide Web and the internet are the same, but they
aren’t. The Web is the most popular way to access online data through hyperlinks and websites; while the
internet, is a vast network of computers and servers on which the World Wide Web operates. The internet
was a tool for scientists, engineers and the military; the web made it accessible to everyone else.
Berners-Lee worked at CERN, where he developed the very first webpage; it went live in August 1991, is
still active, and is probably the world’s first website (bit.ly/2SnA7zy). It was in 1994, however, that the World
Wide Web Consortium (W3C) founded by Tim Berners-Lee set up protocols, guidelines and standards for
the web, and now-familiar terms like TCP (Transmission Control Protocol), IP (Internet Protocol) and HTTP
(Hypertext Transfer Protocol) were born.
The founding philosophy of the Web was for it to serve as a democratizer and equalizer, to empower the
long tail and eliminate monopolies and intermediaries. The Web did solve three big problems for us: an
information problem with search and wikis, a communication problem with email and messenger tools, and
a distribution problem with file-sharing and e-commerce. But it could not address the two big problems that
it was supposed to solve: one of trust and security, and another of disintermediation—its original
philosophy. In fact, the rise of big tech companies has given us intermediaries that are far more powerful
than ever before. They literally own most of our online data and information. In that sense, they own us.
Now comes the Block chain technology.
The reason for the excitement around blockchain is that it is supposed to solve our unsolved problems—
of trust and of inequality—and thereby bring us closer to the original vision of Time Berners-Lee and his
co-conspirators. So, it is not surprising what he said in the Financial Times, as he announced that Sotheby’s
would auction off the original source code of the Web: The NFT project was his “first foray into crypto", but
he saw similarities in his original vision for the web and the philosophy behind the decentralized network
of Ethereum’s blockchain, which underpins most NFTs. It also resonates with his latest project, Solid, which
is designed to give us back control of our personal data. “The blockchain and Solid communities share the
motivations of wanting to empower people," he said, adding that blockchain projects were motivated by
resistance to central control. Much like the open, democratic and decentralized origins of the Web.
Head Office: 127, Zone II, MP Nagar, Bhopal |+91-7676564400| https://www.toprankers.com Page 29 of 36
114. Which of the following conveys the core message discussed in the passage?
(a) The founding of blockchain technology and its application.
(b) The differences and similarities between internet and the World Wide Web and their contribution to the
world.
(c) The original vision of web and its later deviation, only to be brought closer through block chain
technology.
(d) The limitation of web and its failure in providing immunization against trust and security and
disintermediation.
115. Which of the following is true about the World Wide Web?
(a) The web was invented to act as a democratizer and solve the issue of dearth of information.
(b) The web is yet to solve the communication and the information problem.
(c) The web solved the issue of intermediation as per its original philosophy.
(d) The author corroborates the perception of the public that World Wide Web and internet are the same.
116. What does the author mean by stating “They literally own most of our online data and information. In that
sense, they own us.”
(a) Our data is our identity and those who own our data, own us.
(b) Data is the modern currency of exchange in the tech riddled world.
(c) The big companies do not care about data but about us, the users.
(d) Data is the collection of new information and is the costliest on earth.
117. In the last paragraph, the author ends the passage on a __________ note.
(a) Dismal (b) Hopeful (c) Pensive (d) Confrontational
118. According to the passage, why Berners-Lee is excited about the network of Ethereum?
(a) Because the work is a complete imitation of his work.
(b) Because he will be well paid by Ethereum.
(c) Because his philosophy behind creating web matches with Ethereum’s technology.
(d) Because of the centralized network of Ethereum’s blockchain.
Passage (Q.119-Q.123): The French mathematician Pierre-Simon Laplace (1749-1827) believed that the
Universe was a piece of machinery, and that physics determines everything. Napoleon, who had read up
on Laplace's work, confronted him about the conspicuous absence of a creator in his theory. 'I had no need
of that hypothesis,' came the reply. Laplace might have said the same thing about free will, which his
mechanistic universe rendered superfluous.
Since Laplace's day, scientists, philosophers and even neuroscientists have followed his lead in denying
the possibility of free will. This reflects a widespread belief among theoretical physicists that if you know
the initial values of the variables that characterise a physical system, together with the equations that
explain how these variables change over time, then you can calculate the state of the system at all later
times. For example, if you know the positions and velocities of all the particles that make up a gas in a
container, you can determine the positions and velocities of all those particles at all later times. This means
that there should be no freedom for any deviation from this physically determined trajectory.
Consider, then, that everything we see around us - rocks and planets, frogs and trees, your body and brain
- is made up of nothing but protons, electrons and neutrons put together in very complex ways. In the case
of your body, they make many kinds of cells; in turn, these cells make tissues, such as muscle and skin;
these tissues make systems, such as the heart, lungs and brain; and these systems make the body as a
whole. It might seem that everything that's happening at the higher, 'emergent' levels should be uniquely
determined by the physics operating beneath them. This would mean that the thoughts you're having at
this very moment were predetermined at the start of the Universe, based on the values of the particle
physics variables at that time.
Head Office: 127, Zone II, MP Nagar, Bhopal |+91-7676564400| https://www.toprankers.com Page 30 of 36
119. Which of the following statements may have formed the premise for Laplace's argument that 'I had no need
of that hypothesis'?
(a) He was an atheist and did not believe in the existence of a sole creator, after all.
(b) He proved that biological activity at the micro level is literally grounded in the physical shape of
biological molecules.
(c) He might have proved or formulated the existence of all events in the Universe on the basis of physics
and mathematical equations, rejecting the plausibility of free will.
(d) He failed to explain some of the events which were beyond the realms and reach of theories of physics.
120. Which of the following would undermine the argument made in the passage that if you know the initial
values of the variables that characterise a physical system, together with the equations that explain how
these variables change over time, then you can calculate the state of the system at all later times?
(a) Ions are atoms that have become electrically charged because they have lost or gained an electron.
(b) Heisenberg's uncertainty principle introduces an unavoidable fuzziness and an irreducible uncertainty
in quantum outcomes. You might know the value of one variable, such as a particle's momentum; that
means you can't accurately detect another, such as its position.
(c) An apple is suspended from a branch of the tree by a string attached to its stalk. It would thereby have
been turned into a pendulum, because the string constrained its motion. Instead of dropping to the
ground, it would have swung back and forth in a circular arc under the branch, with its state of motion
determined uniquely by its initial position and velocity.
(d) All of the above.
121. Which of the following can we infer from the passage above?
(a) The state of a system is described by what's known as its wave function, which determines the
probabilities of different outcomes when events take place.
(b) At very small scales, theory of Physics underlies what's happening in the world.
(c) The structure of the molecules is truly the secret of life.
(d) Outcomes don't depend only on the equations and the initial data.
122. Which of the following is most likely to be true had theories of physics and mathematical equations failed
to explain some of the earthly events?
(a) Learning and memory offer example of how downward causal effect shapes the underlying physics.
(b) If you seriously believe that fundamental forces leave no space for free will, then it's impossible for us
to genuinely make choices as moral beings.
(c) The power of choice enables physiological systems such as the heart and brain to function in a way
that is enslaved by the lower-level interactions, and choosing the outcomes of the preferred interactions
from a multitude of options have been predetermined at the elementary level.
(d) None of the above.
123. Which of the following most accurately expresses the main point of the passage?
(a) The confounding thing for free-will sceptics is that all outcomes don't depend only on the equations and
the initial data; they also depend on randomization.
(b) In a physically determined world, there is freedom for deviation up to a certain extent.
(c) There are events or things which Physics is unable to answer or reach to an acceptable conclusion.
(d) An understanding of physics sees determinism at work in the Universe.
Head Office: 127, Zone II, MP Nagar, Bhopal |+91-7676564400| https://www.toprankers.com Page 31 of 36
Passage (Q.124-Q.128): Some of us think. Many of us think that we think. And most of us never think of
thinking. Very few use the grand prerogative of the mind, many never think, but they think they do. Very
few of us really think because, to think, we should have the freedom to do so.
We touch the earth with our physical foot; we touch life with our psychological foot. That is why in temples,
we leave our slippers outside. We leave certain coverings that cover our feet and enter the temple barefoot.
In the nakedness, you discover the truth. Shoes cover our feet, and thereby we don’t deeply connect to
God. Our psychological foot, too, is covered by psychological shoes, which are our conditioning, dogmas,
opinions, and beliefs. With the psychological shoes on, we never touch a life.
Very few of us can look at a flower as it is. When somebody looks at a flower, they look at it with the
psychology of likes and dislikes, of should or should not, or must or must not. Therefore, very few of us
really look at the flower; instead, we look at our opinion of the flower- our likes and dislikes.
So, we never see it as it is. Removing the shoe means removing the psychological shoes of our
conditioning, our opinions, and dogmas. When you leave these and innocently look at life, it is then you
see that something very different happens. If you analyze it, you will see that if we are unhappy in life, it is
not because of what is; it is because of our opinion of how life should be. When our view of life conflicts
with our life, this makes us unhappy. This unhappiness is a toxin, a disturbance in our energy field. You
look at situations that you feel should not be, and you get angry. Your anger is often directed as a
scapegoat, not the real genesis of your unhappiness.
124. What does the author talk about when he says ‘the grand prerogative of the mind’?
(a) The illusion of thinking while not thinking in reality.
(b) Utilizing the power of mind to think freely.
(c) Not having the freedom to think.
(d) Actually thinking something/anything at all
125. Why has the author mentioned the psychological shoe?
1. To show that it is deceptive.
2. To show how it covers are true understanding of the world.
3. To show how it distorts our thinking.
(a) Only 1 (b) 1 and 2 (c) 2 and 3 (d) All of the above
126. Based on the information above, which is the most accurate?
(a) We are unhappy because we gauge anger from the wrong prism.
(b) To have an objective clarity about happiness, unhappiness and anything in life, the opinionated shoe
needs to be left out.
(c) The real source of unhappiness and anger lies within us.
(d) The naked soul and the bare feet help connect well with the world.
127. It can be inferred from the passage that the reason of our unhappiness is-
(a) Our opinion is only reason responsible for lack of happiness in our lives.
(b) Consonance in what we think happiness is and what happiness in reality is.
(c) Our psychological shoe, that prevents us from connecting and analyzing things in their natural colour.
(d) Differentiated opinions on life.
128. Statement: ‘Therefore, we never see it as it is.’
The above statement is:
(a) Conclusion statement. (b) Weakening statement.
(c) Strengthening statement. (d) Parallel reasoning.
Head Office: 127, Zone II, MP Nagar, Bhopal |+91-7676564400| https://www.toprankers.com Page 32 of 36
129. In the next few years, molecular biologists will complete mapping the human genome. This means that
scientists will be able to trace each health problem to a particular gene. It will then be possible to fix this
problem by fixing the responsible gene. The hope is that several addictions, such as smoking, alcoholism
and drug abuse, will be curable through gene therapy.
Which of the following assumptions is the above argument based on?
(a) Genetic manipulation is the sole way to cure addictions.
(b) In the future only genetics will matter in the world of medicine.
(c) Addictions are genetically induced, not caused by other factors.
(d) Every human being is addicted to something.
130. One morning after sunrise Deepak while going to school met Raj at road crossing. Raj’s shadow was
exactly to the right of Deepak. If they were face to face, which direction was Raj facing?
(a) South (b) North (c)East (d) West
Directions (Q.131): In each of the questions given below three statements are followed by some
conclusions. You have to take the given statements to be true even if they seem to be at variance from
commonly known facts. Read all the conclusions and then decide which of the given conclusions logically
follows from the given statements disregarding commonly known facts.
131. Statements:
Some songs are not tunes.
All tunes are drums.
Some tunes are sticks.
Conclusions:
I. Some drums being songs is a possibility.
II. All those drums which are tunes can be songs.
(a) if only I follows. (b) if only II follows. (c) if either I or II follows. (d) if both I and II follow.
Direction (Q.132-Q.134): In a family, there are six members- A, B, C, D, E and F. A and B are a married
couple, A being a male member. D is the only son of C, who is the brother of A. E is the sister of D. B is
the daughter-in-law of F, whose husband has died.
132. How F has related to A?
(a) Mother (b) Sister-in-law (c) Sister (d) Mother-in-law
133. How is E related to C?
(a) Daughter (b) Aunt (c) Cousin (d) Sister
134. Who is C to B?
(a) Brother (b) Brother-in-law (c) Son-in-law (d) Nephew
Direction(Q.135): In the question below are some conclusions are given followed by set of statements.
You have to decide from which set of statements the given conclusions logically follow(s) disregarding
commonly known facts.
135. Conclusions:
I. No puzzle is seating.
II. Some puzzles are coding.
III. Some syllogisms are not seating.
Statements:
I. All puzzles are coding. No seating is syllogism. Some syllogisms are puzzle.
II. No coding is seating. All syllogisms are puzzle. Some Puzzles are coding.
III. Some puzzles are syllogism. All syllogisms are coding. No coding is seating.
IV. No seating is puzzle. Some puzzles are syllogism. All syllogisms are coding.
(a) Only Statement I follows. (b) Only Statement II follows.
(c) Only Statement III follows. (d) Only Statement IV follows.
Head Office: 127, Zone II, MP Nagar, Bhopal |+91-7676564400| https://www.toprankers.com Page 33 of 36
SECTION - E: QUANTITATIVE TECHNIQUES
Direction (Q.136-Q.140): Study the following information carefully to answer that follow.
A bank has five different types of accounts, viz A Type, B Type, C Type, D Type and E Type. The total
number of account holders is 2050. 24%of the total accounts are A Types. One-fifth of the total number of
accounts is D Type. 16% of the total accounts are C Types. Remaining accounts are either E Types or B
Types. The number of B Types is 182 more than the number of E Types.
136. What is the ratio of the total number of D Type to the total number of E Type and B Types together?
(a) 2 : 1 (b) 1 : 2 (c) 3 : 4 (d) 7 : 6
137. If 20% of D Type are non-operative, what is the number of D Type which are operative?
(a) 382 (b) 164 (c) 328 (d) 428
138. The number of C Types is approximately what per cent of the total number of A Types and D Type together?
(a) 63% (b) 26% (c) 46% (d) 36%
139. What is the total number of E Type, C Type and D Type together?
(a) 1027 (b) 1157 (c) 1057 (d) 957
140. What is the difference between the total number of E Type and A Types together and the number of B
Types?
(a) 310 (b) 410 (c) 210 (d) 390
Head Office: 127, Zone II, MP Nagar, Bhopal |+91-7676564400| https://www.toprankers.com Page 34 of 36
Direction (Q.141-Q.145): Study the following information carefully answer the questions given below:
In an examination (consisting of two papers Legal and English) total 300 students appeared. Out of that
the ratio of boys to girls is 3 : 2. The number of boys who passed only in Legal is 25% of the total number
𝟑
of boys and this number is 𝟐 of the number of girls who passed only in English. The number of girls who
passed in both the papers is 40/3% of the total number of students and the number of boys who passed
in both the papers is 180% of the number of girls who passed in both the papers. None of the candidate
failed in both the papers.
141. How many girls are there who passed only in Legal paper?
(a) 35 (b) 40 (c) 45 (d) 50
142. The number of boys who passed only in English is what percentage of the total number of students who
appeared in the examination?
(a) 21% (b) 36% (c) 48% (d) 72%
144. What is the ratio of the number of boys who passed in English to the number of girls who passed only in
Legal?
(a) 23 : 8 (b) 25 : 11 (c) 27 : 10 (d) 29 : 15
145. How many students are there who passed at most in one subject?
(a) 172 (b) 178 (c) 181 (d) 188
Head Office: 127, Zone II, MP Nagar, Bhopal |+91-7676564400| https://www.toprankers.com Page 35 of 36
Direction (Q.146-Q.150): Study the given pie chart carefully to answer the questions that follow.
Percent wise distribution of passengers travelling in different trains
Train L
Train M 15%
20%
Train A
13%
Train S
24%
Train Q
19%
Train R
9%
147. If in train R, 34% of the passengers are females and 26% are children, what is the number of males in that
train?
(a) 360 (b) 306 (c) 308 (d) 318
148. Number of passengers in the train Q is approximately what per cent of the total number of passengers in
train A and Train R?
(a) 90% (b) 70% (c) 75% (d) 86%
150. How much more per cent (approximately) is number of passengers there in train M as compare to number
of passengers in train L?
(a) 29% (b) 49% (c) 43% (d) 33%
Head Office: 127, Zone II, MP Nagar, Bhopal |+91-7676564400| https://www.toprankers.com Page 36 of 36
LEGALEDGE TEST SERIES
Part of the Most Comprehensive & Consistently Successful Study Material & Test Series Module, spanning
across both Physical and Online Programs in the entire Country. As a result, LegalEdge was able to engineer
Clean-Sweep-Landslide figures of a handsome 64 Selections under 100 ranks, and a whopping 273 selections
under 500 ranks in CLAT 2021. With AILET being no different, a total of 34 of our students found their way into
NLU, Delhi in 2021. In a nutshell, every second admit in a Top National Law School in 2021 came from the
LegalEdge Preparation Ecosystem.
Head Office: 127, Zone II, MP Nagar, Bhopal |+91-7676564400| https://www.toprankers.com Page 1 of 12
ecosystems such as coral reefs, which provide global partnership of more than forty governments
extraordinary contributions to both biodiversity and more than forty civil society organization with
(about 25 percent of all marine biodiversity across coral reef expertise, have agreed on this framework
about 0.1 percent of the ocean floor) and human that are critical for coral reefs.’
wellbeing (economic and food security for hundreds 11. (b) Option b) is correct as per the last line of the fifth
of millions of people).’ The abovementioned paragraph and the sixth paragraph which gives the
sentence conveys that the important ecosystems are example of Iran and its dependence on the BOS
which provide extraordinary contribution to both technology and how it can be dangerous if any
biodiversity and human wellbeing. country tries to get rid of the technology. 'Critical
Hence, option (d) is correct. Options (a) and (b) are element of dependence' refers to that part of
limited in scope as they provide incomplete dependence of technology, where, if the technology
information. Also ‘only’ in option (b) makes it is not there then many important systems are
redundant. Option (c) takes into account only one disturbed. Hence, the answer is option b). Option a)
aspect and does not mention biodiversity. and c) reflect the advantages of BDS. Option d)
8. (c) Refer to second paragraph, ‘…governments are provides a mere fact, but does not discuss the
failing to meet existing global targets for biodiversity, flipside of BDS.
and that critical ecosystems like coral reefs will be 12. (a) Option a) is correct as the fifth paragraph talks about
altered to the point that the biodiversity they hold, the trick behind the BDS that how it is going to be a
and the services they provide, will be damaged problem in the disguise of a solution. The given
beyond repair.’ paragraph using the example of Iran addresses the
It can be concluded that coral reefs provide cons of the BDS technology and make the country
contribution to human wellbeing (economic and food using the technology highly dependent on it. Option
security for hundreds of millions of people) as well as a) is also the essence of the passage. Option b) is
biodiversity (about 25 percent of all marine an example of how dependence on BDS is tricky.
biodiversity across about 0.1 percent of the ocean Option c) again is more or less addresses the reason
floor). Option (a), (b) and (d) are incorrect as of entrapment. Option d) is irrelevant.
answers to the question stem. Hence, option (c) is 13. (b) The word 'benign' means 'pleasant; not harmful or
correct. severe'. 'Propitious' which means 'giving or indicating
9. (a) A title carries the essence of a topic in brief. The a good chance of success; favourable' is similar in
passage starts with the problems surrounding the meaning to 'benign'. 'Meticulous' means 'showing
coral reefs and then explains how coral reef is getting great attention to detail; very careful and precise',
extinct and at the end talks about global cooperation 'unerring' means 'always right or accurate' and
that can save coral reefs. Option (a) is the only option 'solemn' means 'characterized by deep sincerity'.
which best summarises the above idea. Option (b) is Hence, the answer is option b).
a partial addressal; hence, cannot be the title of the 14. (c) Option c) is incorrect as the given passage does not
passage. Option (c) is a response to the threat posed talk about the whole world but a few regions in the
by the decline of coral reefs and plays an important, world. The passage talks about the extension of the
but partial role, making option (c) unsuitable as a technology but does not anywhere say that it will
title. Option (c) is a guided the action and affect the whole world. The passage mostly talks
investments of the 196 governments that are party to about poor countries. Hence, the answer is option c).
the Convention on Biological Diversity (CBD) that is (a)- The passage talks about the benefits of BOS
a ray of hope. It is the culminating point; therefore, technology at various points, therefore, negating
cannot be the title. Hence, option (a) is correct. option (a). (b)- It is correct as per the latter half of the
10. (d) Refer to first paragraph’s last sentence, ‘Recent fifth paragraph of the passage, which talks about Iran
advances in "big data" and improved remote sensing in particular. (d}- It is correct as per the latter half of
tools show us that collective human impacts are the second sentence of the passage.
leaving fewer places untouched, with only 15% of the 15. (c) Option c) is incorrect as per the second sentence of
Earth's land mass formally protected and global the last paragraph of the given passage which says
biodiversity declining at an unprecedented rate.’ It that BDS also compensate for climate, it means it
can be easily concluded that the role of “big data”, as somehow helps in the case when there is insufficient
per the passage, is to help in showing how collective rainfall and other climate related issues. Hence, the
human impacts are leaving fewer places untouched answer is option c) Option b) is correct as these
and not in the decline of global biodiversity. It, by no countries use BDS. Refer to the lines, ‘Myanmar,
means, is a plan in action. Laos, Cambodia, Indonesia, Thailand, and Uganda
Hence, option (d) is correct. Option (a) is the first line have used BDS for agricultural and infrastructural
of the passage; hence, is true. Option (b) has been purposes.’ Option d) is directly extracted from the
mentioned in the passage. Refer to the lines, passage.
‘…make it clear that governments are failing to meet 16. (b) Option b) is correct as it tells that family is linked with
existing global targets for biodiversity, and that economic, cultural and other spheres. So, this
critical ecosystems like coral reefs will be altered to means that different societies with different political,
the point that the biodiversity they hold, and the economic, and other different features affect the
services they provide, will be damaged beyond character of family and in the same way affect
repair.’ Option (c) finds a mention in the passage. marriage and kinship also. Hence, the answer is
Refer to the lines, ‘Over the last two years, members option b). Option a) is a simple statement that neither
of the International Coral Reef initiative (ICRI), a strengthens nor weakens the argument. Option c)
Head Office: 127, Zone II, MP Nagar, Bhopal |+91-7676564400| https://www.toprankers.com Page 2 of 12
and d) are universal statements that weaken the III: The Irish Free State was the result of the partition
argument. of Ireland. The Ulster unionists "...reluctantly
17. (d) The word 'perpetuate' means 'make (something) accepted partition…" only because they wished to
continue indefinitely'. 'Sustain' which means 'to remain a part of Britain. This shows that the Ulster
cause or allow something to continue for a period of unionists didn't support the partition or the Irish
time' is similar in meaning to 'perpetuate'. 'Discern' Free State. So, option III is true. Hence, the correct
means 'recognize or find out', 'halt' means 'bring or answer is C.
come to an abrupt stop' and 'cease' means 'bring to 24. (d) Belligerent means hostile and aggressive.
an end'. Hence, the answer is option d). Strife means bitter sometimes violent conflict or
18. (b) Option b) is correct as per the last sentence of the dissension political strife.
third paragraph of the given passage which says that Appease means pacify or placate (someone) by
the roles mentioned in the previous sentence is acceding to their demands.
questionable and it is not according to the empirical Gazed means look steadily and intently, especially in
studies across cultures. Hence, the answer is option admiration, surprise, or thought.
b). Option a) is contrary to what is mentioned in the Contested means causing dispute or argument.
passage. The division of roles in nuclear family, is Aggressive means ready or likely to attack or
questionable, not justifiable. Option c) is contrary to confront; characterized by or resulting from
the passage. Option d) is out of scope as it is not aggression. Hence, the correct answer is d.
supported in the passage. 25. (c) The passage states that the only gain from the
19. (a) Option a) is correct as per the second sentence of "plantations" was establishing a community "loyal to
the fifth paragraph of the given passage which says the British crown". Since this was the aim of the
that increase in life expectancy is a contributing plantations, we can say this is one of the reasons
factor which led to increase in joint family. Hence, the behind the colonization of Ireland.
answer is option a). The rest of the options are If the British wished to spread Protestantism, they
incorrect regarding the question stem. would've converted the Catholics, which they didn't.
20. (c) According to the first paragraph, the author conveys The Irish resistance was a result of the colonization,
the idea that the institution of family is something that not the reason behind it.
is common everywhere but it is not the same as it The passage doesn't provide any additional
varies as per the circumstances and society the information on the rest of the options. Hence, the
families are in. Hence, the answer is option c). correct answer is C.
Option (d} misses out on the universal aspect of 26. (d) Going through the passage, we know that the author
'family' as a social institution, therefore, is eliminated. is an ornithologist, on a bird spotting expedition. The
Option a) is incorrect as the passage is not about the journey to Thattekad bird sanctuary was very
importance of families. Option b) is far-fetched as the tiresome and on entering the rest room the photo of
author’s purpose. an unattractive bird did nothing to lift his tired spirits.
21. (d) In the given sentence, we are stating the fraction that After a restful night the author, being a true spirit
the Irish Catholics make out of the total. ‘by’ should ornithologist, is raring to go and hoping to spot the
be used when there is a comparison bin made. ‘a frog mouth as it was a declared endangered species.
third’ is singular so it should be ‘less than a third’. Hence choice D is the correct answer. All the three
Hence, the correct answer is d. options answer the question stem; therefore, option
22. (d) Only options I, II & III are correct. The reasons are as (d) is the answer.
follows: 27. (b) Birders is a synonym for ornithologists and birding is
I: “...the native Irish Catholic population of Ulster was observation of birds in their natural habitat, and this
not converted to Protestantism'' shows that is what birders do. Bird watchers are bird lovers and
Catholicism was not removed at all. The Republic of are zealous about their hobby or profession to a
Ireland that gained independence in 1922, was fanatical degree. Thus, the phrase ‘birding fanatics’
formed by a majority Catholic. So, option I is false. and the answer is as captured by choice (B). Hence,
II: The British went out of their way to establish “...a choice (b) is the right answer Choices (a) and (c) are
base loyal to the British crown…" and not to the not found in the paragraph and rejected. Choice (d)
Pope. Clearly the crown is not under the authority of captures part of the answer and hence, rejected.
the Pope. So, option II is false. 28. (c) Ornithologists or bird-watchers are always one with
III: "Plantations" were a social engineering exercise nature as they observe birds in their natural habitat.
and, thus, can be called a restructuring plan. In this The author and his friends had a long trek for
plan, tens of thousands of English and Scottish spotting birds and an opening with a water body and
Protestants were planted in Northern Ireland. So, absolute silence was the perfect foil for the strain and
option III is true. Hence, the correct answer is D. the author becomes one with the silence and forgets
23. (c) Only options I, II & III are correct. The reasons are as himself for a while. This is captured perfectly in
follows: choice c) and is the right answer Choices a), b), and
I: Ulster is described as “...the northernmost of d) are also true but do not capture the essence of the
Ireland’s provinces" and nothing else. So, option I is scene. Hence, they are rejected.
false. 29. (c) Frog is an amphibian — meaning it can live both on
II: “These fights played out in political debates, the land and in water. It has a wide gaping mouth which
media, sports, pubs – and, often, in street violence.” houses a long cleft tongue capable of capturing prey
This is only possible if the public actively takes part from a distance. The only similarity between the
in the conflict. So, option II is true. amphibian and the bird can be — as evident in the
Head Office: 127, Zone II, MP Nagar, Bhopal |+91-7676564400| https://www.toprankers.com Page 3 of 12
name itself — a wide gaping mouth. Hence choice C 15% has been agreed by 136 countries (including
is the right answer Choices A, B, D are just ramblings India).
and not true. 40. (b) The OECD is an intergovernmental economic
30. (b) The author, his friends and the guide have been organisation, founded to stimulate economic
trekking in the forest for a long time with a hope of progress and world trade.
catching a glimpse of the frog mouth birds. Finally at Founded: 1961.
Pandapara, after wading knee deep in a stream the Headquarters: Paris, France.
guide points out quietly to the author a tree to the left. Total Members: 36.
The author looks hard but finds nothing. Suddenly he India is not a member, but a key economic partner.
sees something blink and he is able to see a pair of 41. (c) With the establishment of the NGT, India became the
frog mouths in perfect symphony with the foliage third country in the world to set up a specialised
and realizes that had one of the birds not blinked he environmental tribunal, only after Australia and New
would have never seen them and even if he had Zealand, and the first developing country to do so.
seen them, they were so motionless that he might 42. (b) NGT specialised body set up under the National
have mistaken them for stuffed birds. Choice b) Green Tribunal Act (2010) for effective and
captures this beautifully and is the right answer expeditious disposal of cases relating to
Choices a), b), d) are far-fetched and can be ignored. environmental protection and conservation of forests
and other natural resources.
SECTION - B : GENERAL KNOWLEDGE/CURRENT AFFAIRS 43. (a) NGT is mandated to make disposal of applications or
appeals finally within 6 months of filing the same.
31. (d) Space reforms approach(ISpA): 4 pillars The NGT has five places of sittings, New Delhi is the
First, the freedom of innovation to the private sector. Principal place of sitting and Bhopal, Pune, Kolkata
Second, the role of the government as an enabler. and Chennai are the other four.
Third, preparing youth for the future. The Tribunal is headed by the Chairperson who sits
And fourth, to see the space sector as a resource for in the Principal Bench and has at least ten but not
the progress of the common man. more than twenty judicial members and at least ten
32. (d) Government is playing the role of enabler so that the but not more than twenty expert members.
private enterprises can come forward and take 44. (b) Decisions of the Tribunal are binding. The Tribunal
charge and not vice-versa. The correct answer is d. has powers to review its own decisions. If this fails,
33. (d) In the 2019 Budget, the government had announced the decision can be challenged before the Supreme
the setting up of a New Space India Limited (NSIL), Court within ninety days.
a public sector company that would serve as a 45. (d) Article 141 of the Indian Constitution provides that,
marketing arm of ISRO (Indian Space Research "The law declared by the Supreme Court shall be
Organisation). binding on all courts within the territory of Indi(a)" In
Its main purpose is to market the technologies Article 141 the expression "all courts" has been used
developed by ISRO and bring it more clients that which creates question that whether Supreme Court
need space-based services. comes under the sphere of all courts.
34. (a) The Indian National Committee for Space Research 46. (a) The global Multidimensional Poverty Index (MPI)
was established by Jawaharlal Nehru under the produced by the United Nations Development
Department of Atomic Energy in 1962, on the urging Programme (UNDP) and the Oxford Poverty and
of scientist Vikram Sarabhai recognising the need in Human Development Initiative measures poverty by
space research. INCOSPAR grew and became considering various deprivations experienced by
ISRO in 1969, within DAE. people in their daily lives, including poor health,
35. (a) The Aryabhata spacecraft, named after the famous insufficient education and a low standard of living.
Indian astronomer, was India's first satellite; it was 47. (b) • 1.3 billion people are multidimensionally poor.
completely designed and fabricated in India and • About half (644 million) are children under age
launched by a Soviet Kosmos-3M rocket from 18.
Kapustin Yar on April 19, 1975. • Nearly 85% live in Sub-Saharan Africa (556
36. (d) The Organisation for Economic Cooperation and million) or South Asia (532 million).
Development (OECD) has announced that a global • More than 67% live in middle-income countries.
deal to ensure big companies pay a Global Minimum 48. (a) Of the 80 countries and five billion people for which
Tax (GMT) rate of 15% has been agreed by 136 there is data over time, 70 reduced MPI in at least
countries (including India). one period, with the fastest changes coming from
37. (a) The countries behind the accord together accounted Sierra Leone (2013-2017), followed by Togo
for over 90% of the global economy. (2013/2014-2017).
38. (d) Kenya, Nigeria, Pakistan and Sri Lanka have not yet 49. (c) • The Multidimensional Poverty Index was
joined the deal. The move is part of an evolving launched by the UNDP and the OPHI in 2010.
consensus that big multinationals are funnelling • MPI is based on the idea that poverty is not
profits through low-tax jurisdictions to avoid paying unidimensional (not just depends on income
taxes. and one individual may lack several basic needs
39. (a) Recently, the Organisation for Economic like education, health etc, rather it is
Cooperation and Development (OECD) has multidimensional.
announced that a global deal to ensure big • The index shows the proportion of poor people
companies pay a Global Minimum Tax (GMT) rate of and the average number of deprivations each
poor person experiences at the same time.
Head Office: 127, Zone II, MP Nagar, Bhopal |+91-7676564400| https://www.toprankers.com Page 4 of 12
• MPI uses three dimensions and ten indicators However, the law does not provide a time-frame
which are: within which the presiding officer has to decide a
Education: Years of schooling and child enrollment defection case.
(1/6 weightage each, total 2/6); 58. (d) Grounds of Disqualification:
Health: Child mortality and nutrition (1/6 weightage If an elected member voluntarily gives up his
each, total 2/6); membership of a political party.
Standard of living: Electricity, flooring, drinking water, If he votes or abstains from voting in such House
sanitation, cooking fuel and assets (1/18 weightage contrary to any direction issued by his political party
each, total 2/6). or anyone authorised to do so, without obtaining prior
50. (b) UNDP was formed on 22 November 1965. The permission.
UNDP operates in 125 counties and is funded As a pre-condition for his disqualification, his
entirely by voluntary contributions from UN member abstention from voting should not be condoned by
states. In addition to meeting national development his party or the authorised person within 15 days of
objectives, its activities increasingly focus on fulfilling such incident.
the Sustainable Development Goals (SDGs), such If any independently elected member joins any
as combating HIV/AIDS and ensuring political party.
environmentally sustainable development. The If any nominated member joins any political party
UNDP is governed by a 36-member executive board after the expiry of six months.
overseen by an administrator, who is third-highest 59. (c) As per the 1985 Act, a 'defection' by one-third of the
ranking UN official after the Secretary-General and elected members of a political party was considered
Deputy Secretary-General. a 'merger'. But the 91st Constitutional Amendment
51. (a) The Supreme Court on Thursday asked the Act, 2003, changed this and now at least two-thirds
government to explain how it zeroed in on the figure of the members of a party have to be in favour of a
of ‘₹8 lakh’ as the annual income criterion to identify "merger" for it to have validity in the eyes of the law.
Economically Weaker Sections (EWS) among 60. (a) Kihota Hollohon vs. Zachilhu (1992)
forward classes of society for grant of 10% In the judgment, the Supreme Court clarified that the
reservation in medical admissions under the all India 10th schedule is constitutionally valid. It neither
quota (AIQ). impinges upon the freedom of speech and
52. (c) The 10% EWS quota was introduced under the expression nor subverts the democratic rights of
103rd Constitution (Amendment) Act, 2019 by elected members.
amending Articles 15 and 16. It also upheld the sweeping discretion available to
53. (c) The amendments run contrary to the constitutional the Speaker in deciding cases of disqualification of
scheme, where no segment of available seats/posts MLAs. However, it also held that Presiding Officer’s
can be reserved, only on the basis of economic decisions of disqualification shall be open to judicial
criterion. The amendments also run contrary to the review.
judgment pronounced in the Indra Sawhney V. Union 61. (c) The Henley Passport Index is the original ranking of
of India 1992 case, that a backward class cannot be all the world’s passports according to the number of
determined only and exclusively with reference to destinations their holders can access without a prior
economic criterion. visa. India has been ranked 90th in the most
54. (a) Thirty years ago, on August 7, 1990, the then prime powerful passport report ‘Henley Passport Index
minister V.P. Singh made a historic decision that 2021’.
changed Indian politics and way of ensuring social 62. (b) The best passports to hold in 2021 are:
justice. The then government decided to implement 1. Japan, Singapore (192 destinations)
the recommendations of the Mandal Commission, 2. Germany, South Korea (190)
and open up reservations for Other Backward 3. Finland, Italy, Luxembourg, Spain (189)
Classes (OBCs) in government jobs 4. Austria, Denmark (188)
55. (d) On 21 July 2021, Karnataka became the first state in 5. France, Ireland, Netherlands, Portugal, Sweden
the country to provide one percent reservation for the (187)
transgender community in all government services. 6. Belgium, New Zealand, Switzerland (186)
The government submitted a report to the High Court 7. Czech Republic, Greece, Malta, Norway, United
in this regard, informing that a notification had Kingdom, United States (185)
already been issued after amending the Karnataka 8. Australia, Canada (184)
Civil Service. The job could be given to males or 9. Hungary (183)
females, from the same category, in case of the non- 10. Lithuania, Poland, Slovakia (182)
availability of transgender candidates. 63. (d) The Passport Index is an online tool. It provides
56. (a) The Tenth Schedule - popularly known as the Anti- insights about visa-free travel option. The Passport
Defection Act - was included in the Constitution via Index includes World Openness score, Visa-free
the 52nd Amendment Act, 1985 and sets the scores and Global mobility scores. All the options are
provisions for disqualification of elected members on correct regarding Passport Index.
the grounds of defection to another political party. 64. (a) India fell down to the 90th position, with its passport
57. (a) The decision on questions as to disqualification on holders allowed to travel visa-free to 58 countries.
ground of defection are referred to the Chairman or India shares the rank with Tajikistan and Burkina
the Speaker of such House, which is subject to Faso.
‘Judicial review’. India was ranked 85th in the January 2021 index’,
(84th) in 2020 and (82nd) in 2019.
Head Office: 127, Zone II, MP Nagar, Bhopal |+91-7676564400| https://www.toprankers.com Page 5 of 12
65. (b) Meanwhile, Afghanistan, Iraq, Syria, Pakistan, and about consent. Option c is incorrect because along
Yemen are among the least powerful. Afghanistan is with dishonest intention, consent and possession
the least among all. should have also been mentioned. Option d is
correct as if a person takes away any movable
SECTION – C: LEGAL REASONING property without the consent of the property holder
even though temporarily and with an intention to
66. (d) a, b and c are incorrect because the passage clearly return it later on, it would amount to theft.
mentions that the 1971 act, nowhere provides for the 73. (c) Option A is incorrect because the question does not
express or implied consent of the husband. That is mention that he was acting in good faith. Option B is
why option a is incorrect as the passage nowhere incorrect because the passage clearly states “the
mentions about integral entity. Option b is also intention must exist that the taker of the property
incorrect because husband’s consent is not needed. intends to cause wrongful gain to one person or
Option c is incorrect because the passage is not wrongful loss to another person.” In this case the act
about the locus standi of husband in termination of of Mohan caused Zohan wrongful loss. Option C is
pregnancy related matters. Option d is correct correct because there was wrongful loss to Zohan
because it provides proper reasoning i.e., damages thus fulfilling all the essentials: Dishonest intention,
should not to be granted since the Medical property moved out of possession, no consent.
Termination of Pregnancy Act does not provides for Option D is incorrect because all elements if theft
the express or implied consent of the husband. have been fulfilled.
67. (d) option a is irrelevant. B is not correct as the passage 74. (d) Option A is incorrect because when Z gave his chain
nowhere talks about family consent. C is incorrect to A the possession shifted from Z to A. So, A can’t
because the act provides for an exception if life is in be held liable for theft. Option B is incorrect because
danger or that is acting in good faith. Option d is the chain was in A’s possession so Z’s consent was
correct because Dr. Madhu acted in good faith to not needed. Option C is incorrect because it is
save Radhika’s life. irrelevant. Option d is correct because the chain was
68. (d) The passage does not make any exception as to the in A’s possession therefore selling it would not
woman being a minor or an adult. It only states that amount to theft.
anyone other than the woman carrying the child has 75. (c) Option A) isn’t the right answer because according
no say on the termination of pregnancy. All the other to the passage organising a peaceful protest isn’t the
3 options given do not find their place in the passage same as motivating anti-government sentiments.
and are mere suggestions. Thus Option D is the Option B) isn’t the right answer because the question
correct answer. does not specify his age, neither the passage talks
69. (d) Correct answer is d because the consent of two about the age playing a factor in the liability. Option
doctors is needed to terminate pregnancy up to 24 C) is the right answer because the passage talks
weeks. Option A is incorrect because medical board about the importance of protest with regards to
decides pregnancy cases after 24 weeks. In the individual rights and the duties of the State. Option
present case Sarika was 24 weeks pregnant. Option D) is wrong because the protests weren’t against the
b is irrelevant. Option c again talks about medical government but against government policy.
board. 76. (d) Option A) is wrong because, going by the facts of the
70. (c) The correct answer is Option C as the passage case the MWA isn’t merely protesting against
clearly mentions that the woan carrying the baby is Government policy but it is rallying to overthrow the
the only one who has a say in the termination, thus government. Option B) isn’t correct because the
John and Amy cannot make Bindu terminate the wording of the option is supposed to confuse us
child. Option A and D are incorrect owing to the same about the interpretation of the passage by
logic. Option B is incorrect as even if the child has introducing “collective interest” in place of “protesting
abnormalities, Bindu cannot be forced to terminate against government policy”. Option C) is an
the pregnancy by anyone. irrelevant choice. D) therefore, by eliminating the rest
71. (d) Option A is incorrect because section 378 talks about of the choices we can mark option D) as our right
possession and not ownership. Option B is incorrect answer.
because the answer is incomplete and lacks legal 77. (b) Answer A) is wrong because it is factually incorrect,
reasoning. Aman is guilty of theft because he took the article mentioned in the passage is 51-A(g) and
the suit from tailor’s possession without his consent. not 52-A(g). Option B) is correct because it properly
Option C is incorrect because there was dishonest explains the underlying principle of the passage.
intention as Aman did not pay tailor his due and Option C) is the incorrect choice as it twists the
simply took away the suit. Option D is correct meaning of the said principle, for being against
because when Aman gave the suit to the tailor the government policy isn’t exactly the same as being
possession was transferred to him (tailor). Thus, against the government. Option D) is incorrect
Aman with a dishonest intention took the movable because we cannot predict or guess what Dholu
property out of the possession of the property holder would do in the future, it’s a vague option.
(tailor) without his consent. 78. (c) We arrive at Option C) by the process of elimination.
72. (d) option a is incorrect because it does not matter Option A) can be eliminated as going by the facts of
whether the file was in the office or not. The main the case, we cannot conclude that the content being
point is whether it was moved out of possession aired is inflammatory. Similarly, we eliminate Option
which is true in this case. Option b is incorrect B) and Option D)
because the answer is incomplete. It does not talk
Head Office: 127, Zone II, MP Nagar, Bhopal |+91-7676564400| https://www.toprankers.com Page 6 of 12
79. (a) Read the Question clearly, it asks for an option that 87. (a) Option A is correct because his remarks openly
is NOT in contravention to the Fundamental Duty states making allegations against the judiciary or
(FD) mentioned in the above passage (duty to judges and attributing motives to judgments hence
protect the environment), thus we are supposed to this amounts to scandalizing the Court of judges,
find an option that is not against the FD but one that thereby undermining people’s confidence in the
withholds its interpretation. Option A) is the correct administration of justice and bringing the Court into
answer as its the only option that isn’t talking about disrepute. Such disrespect tantamount to criminal
direct damage done to the environment, instead it contempt. Option B is incorrect because freedom of
has a positive attribute. Option B), C) and D) have speech and expression is accompanied with
negative attributes and cannot be exceptions to the reasonable restrictions which do not permit
duty one has toward the environment and nature. defamation/contempt of Court. Option C is incorrect
80. (d) Option d is correct as the passage clearly mentions it because his remark certainly raises serious
the individuals are adults not even the parents can questions about the judiciary and tarnishes its
object, making Option C incorrect. The passage reputation which is barred as per the passage.
does not talk about any consenting/legal age, it only Option D is incorrect because the facts do not state
mentions the term ‘adult’ which in ordinary parlance this.Court
means above 18 years of age, which they were, 88. (b) Passage clearly states that fair criticism on merits of
making Option A and B wrong. judicial order after case is heard and disposed of
81. (b) Option b is correct. Since atheism is also a belief and does not amount to contempt of Court. Here in this
not the lack thereof, the same treatment would apply. case the Court rejected Mr. Shashank’s PIL without
Thus, option b is correct. Moreover, since they are of finding any merit in it, thereby meaning it was a fair
consenting age and their rights are protected under criticism and does not amount to contempt of Court.
the constitution insofar as practice and worship, Hence option A is incorrect. Option C &D are
extending to marrying, options c and a are incorrect. incorrect for it is a fact-based answer and not related
82. (b) Option B is correct. The passage clearly mentions to the passage. Option B is correct because any
two individuals who claim to be in love with each citizen can comment or criticize the judgement of the
other and are major then nobody, not even their Court and therefore it would not be treated as
parents, could object to their relationship. There is no contempt of Court, if that comment is based on the
exception made for homosexual couples in the merits.Court
passage. So if we apply the same reasoning in the 89. (c) Option A is incorrect because his remarks were not
present situation, only Option B stands true. scandalousfor he is commenting onto the merits of
83. (a) Option A is correct. The passage mentions the the case and the passage clearly states that fair and
granting of protection with the specific mention of the accurate reporting of judicial proceedings will not
individuals being adults. Thus, an interpretation can amount to contempt of Court. Option B is incorrect
be drawn that if they are not adults, the court can becauseit is out of the scope of the passage,
rightfully deny them protection for marriage and the passage nowhere states about innocent publication,
parents can object the relationship, making Option D distribution of matter and reasonable and fair
incorrect. Option B and C are irrelevant to the criticism of judicial acts and comments on the
present factual situation. administrative side of the judiciary.Option D is
84. (d) Option d is correct. Since the passage does not incorrect because the last line of passage states “Nor
mention ant articles in relation to approaching the is any fair criticism on the merits of a judicial order
court, the 3 options even if correct would fall out of after a case is heard and disposed of.”
the scope of the passage. 90. (a) Option A is correct because his statements are
85. (c) Option c is correct. The registrar, bestowed with prejudices, which interfere or tend to interfere with
authority to register the marriages so envisioned the due course of any judicial proceeding as he was
under his authority. This translates to him having and vilifying all the judges of the High Court. Option B is
bearing the liability arising out of his duties, and thus, incorrect because freedom of speech and
options A and B are incorrect. Moreover, option D expression is accompanied with reasonable
lacks any reasoning whatsoever and thus, is restrictions which do not permit defamation/
incorrect. contempt of Court. Option C is incorrect because this
86. (a) Option A is correct because his remarks made option does not explain the fact as to how his
against the Judge are scandalous and can prevent comments amount to fair criticism on merits of
the course of justice by interfering with the proper judicial order. Option D is incorrect because this is a
administration of justice, hence it amounts to fact-based answer and not related to the passage. .
contempt. Option B is incorrect because even 91. (c) Option A is incorrect because the act is applicable to
Fundamental rights are not absolute, there are any person who is inside the territory of India, line 1
always accompanied with certain reasonable of passage clearly provides for it. Option B is
restrictions. Option C is incorrect because no where incorrect because NDPSexception up till 25 kgs is
the questions states what had happened or what only for bhang, here the facts do not mention about
were the true facts. Option D is incorrect as it is bhang being in use. Option C is the correct answer
clearly visible the tweet had a malicious intent. It as the passage clearly states “NDPS Act is to
states that the CJI is enjoying a ride on an expensive prohibit “the manufacture, production, trade, use,
bike which clearly questions the conduct of the etc. of narcotic drugs and psychotropic substances.”
judiciary. Thus, even a minimal usage would amount to
punishment. Option D is incorrect because the
Head Office: 127, Zone II, MP Nagar, Bhopal |+91-7676564400| https://www.toprankers.com Page 7 of 12
answer is not in line with the reasoning provided in 97. (c) Passage clearly states that in consumer cases arises
the passage. for deficiency of services ‘employed’, here in this
92. (b) Option A is incorrect because the reasoning provided case the services provided by Mr. Water were not
is wrong, passage nowhere states provision is employed by Mr. Vex, rather Mr. Water offered to
applicable only to citizens. Option B is the correct help Mr Vex, thus he can not be held liable. Thereby
answer as in the case of Sami v. State of UPCourt making option A & D incorrect. Option B is incorrect
categorically held consumption of bhang uptill 25 kgs for it not connected with the passage. Thus, making
does not lead to conviction. Option C and D are option c as correct.
incorrect because as stated in the case Sami v. State 98. (c) Option A is not correct as given above in the passage
of UP bhang if consumed within permissible limitisn’t “The onus of proof that there was deficiency in
not punishable. service is on the complainant.” here Baburao
93. (c) Option A is incorrect because refer to NDPS Act is to Ganpatrao Apte is the complainant therefore the
prohibit “the manufacture, production, trade, use, burden of proof is on him. Thereby making option B
etc. of narcotic drugs and psychotropic substances. & D also incorrect. Option C is correct as given above
Option B is incorrect because Bhang without THC is in the passage “without any proof of deficiency, the
not covered whereas ACID was trading the one opposite party cannot be held responsible for
which contained THC. Option C is the correct deficiency in service” therefore Mr. Baburao
answer. (Refer to Sami v. state of UP where it has Ganpatrao Apte could not claim compensation
been explicitly held that consumption as well as without providing any material evidence and proof of
possession of psychotropic substance which Mr. Raju’s deficiency of service.
contains THC without doctor’s prescription is illegal.) 99. (a) Option B is not correct as whale Ltd. has failed to
Option D is incorrect because this option is narrow in meet the product specifications which in itself are a
scope and does not talk of distribution. imperfection and inadequacy in the quality or
94. (b) Option A is incorrect because the passage nowhere deficiency of service. Option C is also not correct asit
states what is the general principle of law; Option B is a fact based and not connected with the passage.
is the correct answer. (Refer to section 54 of NDPS .thus is bound to maintain the product specification.
act given in the Question) Option C is incorrect Option D is incorrect for it is narrow in scope and
because the reasoning is not mentioned in the does not specifically lay down what was the fault on
passage. Option D is incorrect because section 54 their whereas option A clearly states the fault.
clearly states “under this Act, it may be presumed, Therefore, option A is the correct answer.
unless and until the contrary is proved, that the 100. (b) Option A is correct as given above in the passage
accused has committed an offence”. Therefore, “The onus of proof that there was deficiency in
option b is correct. service is on the complainant.”
95. (a) Option A is the correct answer as the passage clearly Option b is incorrect for the passage clearly provides
provides that production of narcotic drugs is in para 2 If sample retained by the appellant at the
prohibited except by medical expert. Option b & c are time of consignment is materially different than what
incorrect for it is a fact-based answer not related to was certified by the appellant, the burden of proof
the passage. Option d is incorrect as it is merely would not shift on the appellant
making an assumption not provided for in passage Option c is true whilst it cannot drawn from the
or facts. passage directly, however the passage states that
96. (d) The pre requisite required for proving deficiency in deficiency of service can be alleged only and if there
service has to be necessarily accompanied with is attributing fault, imperfection, shortcoming,
attributing fault, imperfection, shortcoming or inadequacy in quality of the service provider etc,
inadequacy in the quality, nature and manner of thereby meaning, he is to act prudently and in a
performance which is required to be performed by a reasonable manner.
person in pursuance of a contract or otherwise in Option d is incorrect for it is true and clearly provided
relation to any service. The burden of proving the for in para 1.
deficiency in service is upon the person who alleges 101. (d) The passage clearly states visa is being granted to
it. Here in the facts given no where the facts mention facilitate the stay of Afghan nationals fleeing
that there was any deficiency on the part of the Afghanistan for six months in India. There is no
hospital, in fact as a matter of fact, it is provided that clarity yet on what will happen after this validity
Doctors isolated the patient and treated him period expires. Hence, they are not liable to
according to the Covid guidelines of ICMR despite all punished or be prosecuted. Thereby making option
that he died. Thereby making Option Aincorrect A incorrect. Option B is incorrect for it is not related
Option B is also not the appropriate choice as it to the passage. Option c is incorrect for the passage
totally blames the condition of the patient for his clearly states in para 3 that if the visa is granted for
death and absolves the hospital without giving any a period of stay exceeding 185days, then the visa will
adequate reasoning further this option is a fact based be non-extendable and non-convertible to any other
answer and not related to the passage; Option C is type of visa. Here in this question the visa was
incorrect and irrelevant here for the question is only granted for a period of 6 months thereby meaning for
with respect to hospital’ liability and not with regards a period of less than 185 days. Thus, option D is
to Mr. Haldiram. Option D is correct as facts also correct.
suggest that the hospital treated him with the 102. (a) Option A is correct the passage clearly provides the
approved guidelines of ICMR for covid and no facts provision for e visa in the said question is for
suggests any imperfections or fault on their part. facilitating stay of Afghan nationals fleeing Afghan for
Head Office: 127, Zone II, MP Nagar, Bhopal |+91-7676564400| https://www.toprankers.com Page 8 of 12
6 months in India, here in this case facts are clear and still consider themselves the authority on these
that he is a UK resident, thereby provisions of this topics. Option b) is also correct as the author
visa regime won’t be applicable on him, and his visa mentions that monetarism's assumptions had
will be rejected. Thereby making option b & d become accepted as common sense until the 2008
incorrect. Option c is incorrect it merely states a fact recession. Option c) is correct as it is mentioned in
but does not give any reasoning. the first paragraph that economics is trying to solve
103. (c) Passage clearly states that India does not have a problems that do not exist. Therefore, option d) is the
refugee policy and grants shelter to foreigners facing answer. It is not an inference because it is already
persecution in their home country on a case to case mentioned in the last paragraph.
basis. Here in this case the passage whilst is about 108. (c) Option a) is incorrect as this was an assumption of
Afghan nationals fleeing from their country, it does monetarism in the 1980s and not something that the
state on case to case it can grant shelter, further as author deduces. Option b) is also incorrect as the
the passage is not talking about illegal immigrants last paragraph mentions that modern economists
hence, we cannot comment on it. Thereby making have failed in finding solutions to these problems.
option b and d incorrect.Option A is incorrect as it is Option d) is also incorrect as the author disapproves
against the facts as Sharbat Gula is Afghani and of this shift of economics towards mathematics.
Ashraf Ghani Bangladeshi. Option d is incorrect for Hence, option c) is the answer which is mentioned in
the passage talks about nothing in relation to right to the second paragraph of the passage. Refer to the
dignity, hence can be ruled out. lines, ‘This continues to be the case, despite the fact
104. (d) The pre requisite laid by the passage for a Afghan that, since the 2008 recession, central banks have
national seeking refuse in India is by applying for e- been printing money frantically in an attempt to
Emergency X-Misc Visa for 6 months, and no other. create inflation and compel the rich to do something
Here in this case Ram has forged document to enter useful with their money, and have been largely
in India which by no means is lawful within the unsuccessful in both endeavours.’
meaning of the passage, hence he will not be 109. (a) Refer to the lines, ‘The deliberate or unintentional
allowed to enter in India. Option b for it is making spread of misinformation, despite capturing
assumptions that done crimes in Afghanistan widespread public attention, remains as rampant as
including forging passport which are not provided for ever…’ and ‘Misinformation is a long-standing
in factual matrix. Option c is incorrect for it is not problem that demands long-term, sustainable
addressing the question of whether he will be solutions as well as short-term interventions.’ Option
allowed to enter India. Option A is incorrect for it is a (a) is true as the author states that misinformation
fact-based answer having no relevance to the has been existing for a long time.
passage. Option (b) may be true but cannot be ascertained
105. (c) Legal reasoning questions are always answered from the text of the passage. The author is strictly
basis the principle/ passage. Here in the passage,it against the spread of misinformation. Plus, morally it
is clearly provided if an Afghan national is provided is not advisable that a bitter truth of the society be
a visa its is for the purpose of facilitating their stay, hidden and a sugar-coated lie be told around. It
here as the facts irrespective of the fact that she is a actually worsens maters, especially during pandemic
Muslim she will be granted the stay for she has times.
procured the visa. Furthermore, CAA bars citizen, Option (c) is incorrect and can be sourced from the
Shyra has seeked refugee not applied for citizenship last paragraph of the passage. The author puts the
of the country. Thus, she will be allowed to stay, onus of fighting misinformation both on the
thereby making option b and d incorrect. Option a is institutions as well as the people. Refer to the lines,
incorrect for it is out of scope the passage, passage ‘It hasn't been enough. If we're to stop
does not discuss the concept of equality. misinformation and its insidious effects, we need to
radically expand and accelerate our counterattacks.
SECTION - D : LOGICAL REASONING It will take all sectors of society: business, non-
profits, advocacy organizations, philanthropists,
106. (b) The author through the second paragraph wants to researchers, governments, and more.’
communicate that the assumptions of the 110. (d) Option (a) won't affect the author's claims as the
economists and the actions of the governments author believes that misinformation is a menace that
since the 2008 crash still continue despite yielding the world is fighting. Thus, this option will strengthen
no success. According to the author, we should have the author's viewpoint.
learnt our lessons well and should have looked at Option (b) can't weaken the author's arguments
fresh alternatives for better economic understanding because the author has not mentioned in the
and sustenance. Option b) best reflects the thoughts passage as to who is spreading the most
of the author. misinformation. He is much more concerned about
Option a) is incorrect as the passage doesn't mention how to tackle this menace. Thus, this statement has,
anywhere that we cannot do anything to improve the in fact, no bearing on the author's argument.
situation. Option c) is also incorrect as it only covers Option (c) can't weaken the author's argument. It has
an example from the passage and not the main point. simply no relevance to the author's argument.
Option d) is incorrect as the author talks about much In light of the last paragraph of the passage, it can
more than teaching. Hence, option b} is the answer. be inferred that the author emphasizes that all and
107. (d) Option a) is correct as the passage lists the problems sundry should come forward to counterattack the
economists are supposed to solve but fail to do so problem. But if the major issue is organizations and
Head Office: 127, Zone II, MP Nagar, Bhopal |+91-7676564400| https://www.toprankers.com Page 9 of 12
the people are behind eradicating misinformation, option (c) is the correct answer. Option (a) is close,
then this assertion is false. Therefore, Option (d) is but does not take into consideration of the reason
the correct answer. behind its origin. Option (b) and (d) serve as
111. (c) Statement 1 is an instance of an unintentional introductions and are incomplete as the core
spreading of misinformation by an innocent person. messages.
While Steve spreads it deliberately, even though it is 115. (a) Refer to the third part of the passage. The passage
a prank, it is misinformation; Tony spreads it very clearly states “The founding philosophy of the
unknowingly. The author has mentioned this in the Web was for it to serve as a democratizer and
first line. Thus, statement 1 is mentioned. equalizer, to empower the long tail and eliminate
Statement 2 is an instance of a person leaving social monopolies and intermediaries. The Web did solve
media after getting frustrated from all the three big problems for us: an information problem
misinformation. Though the author mentions people with search and wikis, a communication problem
finding it difficult to believe everything on the internet, with email and messenger tools, and a distribution
he doesn't mention people quitting social media problem with file-sharing and e-commerce. But it
altogether. Thus, statement 2 is not mentioned. could not address the two big problems that it was
Statement 3 highlights how social media undertakes supposed to solve: one of trust and security, and
a short-term measure to fight the problem of another of disintermediation—its original
misinformation. The author has mentioned it in the philosophy.” Which make option (b) and (c)
passage. Thus, option (c) is the answer. incorrect. Option (d) is contrary to what is mentioned
112. (c) To arrive at the rationale of the answer, we need to in the passage. Refer to the line, ‘A lot of us think that
carefully read the question stem that demands the the World Wide Web and the internet are the same,
most significant agent. ‘Most’ means that others also but they aren’t.’ Corroborate means to confirm the
play their part but one that is most effective as an given information, which is not the case in the
agent. In expression like ‘most’, there will be more passage.
than one close options. One has to choose the 116. (a) We share our information with the technology
closest to what has been demanded as the answer. companies and they have control over those
Statement 1 is not correct as the people need to be information. So, option A is the most apt choice.
careful not to spread misinformation; the reason Rest of the sentence are not related to the passage.
given in the option is insufficient. Also, it is not the Thus, option (a) is the correct answer.
most significant as the process is slow, as mentioned 117. (b) The meaning of the following words:
in the passage. The reason is that a change in Dismal: filled with hopelessness.
people will help tackle the issue in a better way, but Hopeful: feeling or inspiring optimism about a future
in the long run. event.
Statement 2 is correct as the author points out that Pensive: thinking in a quiet way, often with a serious
the companies, though, not solely responsible expression on your face.
agents for this menace, have a better chance of Confrontational: challenging and argumentative.
curbing the menace as they have the technical and The passage ends with stating that blockchain
logistical aids. Instant blocking of miscreants technology is going to take back the autonomy which
spreading the information and a sound warning will web intended to when it was invented, that is a space
be a deterrent. which is free of intermediaries. The passage ends
Statement 3 is incorrect as the actions by with a solution at hand. The author leaves the
government will bring about a change, but in the long passage on a positive note. Thus, option (b) is the
run. Political fraternities have influential people. They correct answer.
can educate the people far better as not to spread 118. (c) The passage states that “: The NFT project was his
misinformation than anyone else, is certainly true but “first foray into crypto", but he saw similarities in his
they cannot be the most significant agents. original vision for the web and the philosophy behind
Thus, option (c) is correct. the decentralized network of Ethereum’s blockchain,
113. (b) The author in the last paragraph clearly provides that which underpins most NFTs.” Thus, option C is the
the solutions suggested and implemented before is correct answer. Option (a) is incorrect as the phrase
not enough. Further, the battle cannot be won by ‘complete imitation’ finds no mention in the passage.
only investing in the supply side of the information. Option (b) does not find support in the passage.
Therefore, the correct answer is Option (b). Since, Option (d) is contrary to the passage. Read the quote
the main statement is explicitly mentioned in the above.
passage, it cannot be probably true; therefore, option 119. (c) (a}- It can't be derived from the passage as there is
(a) is incorrect. The main statement does not state no evidence implied or explicitly mentioned which
anything that defies the content given in the points towards his belief in God. {b}- It is out of scope
passage, makes option (c) and (d) incorrect. as the context in which the argument was made has
114. (c) The passage talks about how web had certain vision, no relation with biological activity at micro level. (d}-
as the passage states “serve as a democratizer and This would undermine the premise made by Laplace
equalizer, to empower the long tail and eliminate as it would open doors for the possibility of existence
monopolies and intermediaries.” However it got of a creator. (c}- It is correct as the premise is
deviated and finally the block chain technology might possibly based on the fact that Laplace might have
bring “closer to the original vision of Time Berners- proved or formulated the existence of all events in
Lee and his co-conspirators.” Thus, option C tile Universe on the basis of physics and
encompasses the theme of the passage. Thus,
Head Office: 127, Zone II, MP Nagar, Bhopal |+91-7676564400| https://www.toprankers.com Page 10 of 12
mathematical equations. Hence, (c} is the right 125. (d) The passage clearly states how putting the shoes on
answer. covers us with opinions and dogmas and then when
120. (b) (a)- It is irrelevant in the context of the question as it we see things, they get shaped by our vision. Our
talks about the definition of ions. (c)- It talks about a vision of anything isn’t a true picture and that the
change brought about by suspending the apple from source of our happiness isn’t the reality but how the
string and explaining its motion. This doesn't shoes presented things in front of us. All of the
undermine the argument asked in the question. {b}- following reflect the psychological shoe.
It talks about not accurately determining all the 126. (b) To have an objective clarity about happiness,
variables at any given point of time, which certainly unhappiness and anything in life, the opinionated
undermines the arguments of Knowing initial values shoe needs to be left out. The opinionated shoe here
of all the variables involved. Hence, {b) is the right is the psychological shoe we put on, and such a shoe
answer. Since option (b) is the correct answer, option prevents us from having a purer connect with reality,
(d) is ruled out. life, things, and happiness in general. When we have
121. (b) (a), (c} and {d)- they are additional information which that shoe on, our views are fared by the opinion we
are essentially out of the scope of the given passage hold of them. For example- someone who doesn’t
and therefore, can't be said to be inferred. Option (a), like the colour red will shape his view of red rose on
by stating different outcomes suggests a contrary it (with shoe on) but that’s a personally inclined
view to Laplace. Option (c) is too far-fetched to be choice as per the author. Option (a) is vague. The
inferred. It must take into account various intertwined phrase ‘wrong prism’ does not provide clarity. Option
factors before inferring something that abstract. (c) is close, but not the MOST accurate. Option (d) is
Option (d) is contrary as an inference. {b)- it can be far-fetched.
inferred from the last few sentences of the passage 127. (c) Refer to the lines, ‘If you analyse it, you will see that
which provide the example of human body (made up if we are unhappy in life, it is not because of what is;
of small particles electrons, protons and neutrons} it is because of our opinion of how life should be.
and how it operates. Hence, {b} is the right answer. When our view of life conflicts with our life, this
122. (d) The question stem attempts at weakening the makes us unhappy.’ The correct option is (c)-our
content of the passage by introducing a question that psychological shoe, that prevents us from
challenges the essence of the philosophy that if you connecting and analysing things in their natural
know the initial values of the variables that colour and not through gauging the true meaning.
characterise a physical system, together with the Option C is the gist of the passage which also
equations that explain how these variables change supports the question stem. In option (a), the author
over time, then you can calculate the state of the does not use ‘only reason’ as the answer. Option (b)
system at all later times. It means that a latter event is contrary to the passage as it is a conflict and not
is not a deviation from the trajectory. It is expected consonance (harmony) in our perception and reality
and explained for. that is the root-cause of unhappiness. Option (d) is
All the options given here are either agreeing or are out of scope.
far-fetched and way outside the scope of the 128. (a) conclusion statement. The main argument is that we
passage. Option (a) answers the facts given in the never see a flower as it is. The statement given can
passage and is not aligned with the question stem. be logically deduced as our vision is shaped by our
Option (b) is more of ‘thinking out loud’ based on the opinion, and the psychology of our likes and dislikes.
content of the passage. Option (c) corroborates with 129. (c) As we are now aware, an assumption can be tested
the content of the passage. So, no option is likely to by checking if the argument still holds if the opposite
be true in case of the situation mentioned in the of the assumption is assumed to be true. If the
question as options are unrelated to the situation opposite of (C) is true, then addictions are caused
given. Hence, (d) is the right answer. due to non-genetic reasons, meaning that the
123. (d) (a)- It is out of scope of the passage as it talks about applications of mapping the human genome cannot
free-will sceptics, which isn't discussed in the cure addictions such as smoking, alcoholism and
passage and can't be the main theme of the drug abuse. Only negating choice (C) invalidates the
passage. Also, it is contrary to the passage. {b}- It is conclusion. Hence, C
incorrect as per the last sentence of the second 130. (b) North. In morning the shadow falls towards the west.
paragraph of the passage. (c)- There is no evidence Raj’s shadow falls to the right of the Deepak. So Raj
or example quoted in the passage which points is facing North.
towards this argument; in fact, all the events quoted 131. (d) if both I and II follow.
has an explanation in Physics and is well The basic diagram for the given statements is:
determined, which is mentioned in option {d} and is
the gist of the passage. Hence, (d} is the right
answer.
124. (b) Using the power of the mind to think freely. The mind
is entitled to think and shape views and opinions
based on its power of free thinking. Not thinking with
pre-determined conditionings. By calling it the grand
prerogative, it is highlighted that such a privilege
exists with humans and great ideas, opinions etc
come from it. Rest of the options do not reflect the
correct interpretation of the phrase. The possible diagram for both conclusions I and II
is:
Head Office: 127, Zone II, MP Nagar, Bhopal |+91-7676564400| https://www.toprankers.com Page 11 of 12
Hence, both I and Ii follow.
Hint (Q.132-Q.134) We have finally reached in the
position of completing our tree diagram by Total = 300
incorporating the final information which is about F Bays : Girls = 3 : 2
who is a female since her husband had died. And B Boys = 180, Girls = 120
is her daughter-in-law. Therefore, she’s the mother 141. (d) Girls who passed only in Legal paper: 50
of A and C. 142. (a) Reqd % = 63 × 100 /300 = 21 %
132. (a) is the correct answer. 143. (d) Total students who passed in Legal = 45 + 50 + 40 +
133. (a) is the correct answer. 72 = 207
134. (b) is the correct answer. 144. (c) ratio = 72 + 63 /50 = 135/50 = 27/10 = 27:10
135. (d) is the correct answer. 145. (d) Students who passed at most in one subject = 45 +
I. True II. True III. True 50 + 30 + 63 = 188
Explanation (Q.146-Q.150):
146. (d) Number of passengers in train S = 24% of 10000
Number of passengers in train M = 20% of 10000
Number of passengers in train L = 15% of 10000
(24%+20%+15%)𝑜𝑓 10000
∴ Required average =
3
59% 𝑜𝑓 10000
= = 1966.66 = 1966
3
147. (a) Number of Passenger’s in train
9
R = 10000× = 900
100
100−34−26 900×40
SECTION - E : QUANTITATIVE TECHNIQUES ∴ number of males = 900 × = = 360
100 100
148. (d) Percentage of people in train Q=19%
Explanation (Q.136-Q.140): Percentage of people in trains A and R together
Number of Savings Accounts = (13+ 9)% =22%
19
= 24 × 2050 /100 = 492 ∴ Required percentage = ( × 100) % = 86%
22
Number of D Type = 2050 × 1 /5 = 410 Note: As we have already discussed, we don’t need
Number of C Types = 16 × 2050 /100 = 328 to take actual values here. We can find the required
Number of E Type and B Types = 820. percentage by taking only their percent terms also.
Number of B Types = No. of E Type + 182 149. (d) Train M has second highest number of passengers
E Type + B Types = 820 as it has second highest percentage.
or E Type + E Type + 182 = 820 150. (d) Percentage of passengers in train M =20%
or, 2 E Type = 820 - 182 = 638 Percentage of Passengers in train L = 15%
E Type = 638/2 = 319 20−15
B Type = 319 + 182 = 501 ∴ Required percentage =( × 100) %
15
136. (b) Reqd ratio = 410/820 = 1:2 𝟓𝟎𝟎
= % ≈ 𝟑𝟑%
𝟏𝟓
137. (c) Number of non- oprative accounts = 410 × 20 / 100
= 82
Number of accounts which are operative = 410 – 82
= 328
138. (d) Reqd% = 328 × 100 /902 = 36.36 ≈ 36 %
139. (c) Total number of E Type, C Type
and D Type = 328 + 319 + 410 = 1057
140. (a) Difference = 319 + 492 – 501 = 811 - 501 = 310
Explanation (Q.141-Q.145):
Head Office: 127, Zone II, MP Nagar, Bhopal |+91-7676564400| https://www.toprankers.com Page 12 of 12